Você está na página 1de 72

Board Review Biological width Pedo: 0-2mm normally; 0-4mm if next to an erupting permanent tooth or exfoliating primary tooth

How much for max NO: flow rate 4-6 L/min; in children, increase by 10% increments, maintenance is usually at 30% Orofacial Clefts Cleft Lip: 25%, 4-5 weeks in utero o defect between medial nasal process and maxillary process o unilateral 80%, bilateral 20%; 1:1000 births Cleft palate: 25%, 6-8 weeks in utero o lack of fusion between palatal shelves o 1:2000 births Both 50% Ectodermal dysplasia: "heritable conditions in which there are abnormalities of two or more ectodermal structures such as the hair, teeth, nails, sweat glands, cranial-facial structure, digits and other parts of the body." Congenitally missing teeth, peg shaped, or pointed, defective enamel Also have frontal bossing, thin brittle hair/nails, light skin, cannot perspire cannot regulate body temperature Bleeding values: Bleeding: 1-3 minutes PT: 10-13 seconds; extrinsic pathway INR: 0.8-1.2; (INR=PTtest/PTnormal) o in patients on anticoagulation therapy- 2.0-3.0 PTT: 25-29 seconds; intrinsic pathway Platelet count: 150k-450k / L o Oral agents that alter/suppress platelet function include: aspirin, clopidogrel, cilostazol, ticlopidine, and prasugrel

Bone graft Autograft same person; from intraoral sites (maxillary tuberosity marrow, osseous coagulum, bone blend); also iliac bone Allograft o uDFDBA osteoconductive (outside cells penetrate scaffold and form new bone) o DFDBA osteogenic (contanes MMPs that induce formation of new bone) Xenograft BioOss anorganic bovine-derived bone osteoconductive Cosmetic dentistry Hue the color Chroma the purity/saturation of the color; high rich, low dull Value lightness or darkness of the color; light tint, dark shade

Class III cavity in two proximal teeth: Drill: BIG to small; Fill: small to BIG Local anesthetic in children: 4.4mg/kg = maximum recommended dose of anesthetic kg X 4.4mg/kg = Xmg max %anes x 10 x 1.8 = Xmg/cartridge Xmg max / Xmg/cartridge = max # of cartridges ANUG caused by fusiform bacilli (spirochetes) Painful, bleeding gingiva, blunted papillae, pseudomembrane, fetid breath, high fever Tx: remove pseudomembrane, calculus, Abts if lymphadenopathy/fever, CHX; re-eval in 1-2 days (more debridement) then in 5 days for another re-eval Herpetic gingivostomatitis within 3 days of onset: treat with Acyclovir 15mg/kg 5 times per day for 7 days All patients: palliative care: plaque removal, systemic NSAIDS, and topical anesthetics Contagious when vesicles are present Ameloblastoma adult, odontogenic, molar region, benign, aggressive, recurrent, external swelling Mucus retnsion cysts blockage of salivary duct by sialolith (salivary stone); blueish; on palate, Buccal mucosa, upper lip Called a ranula if on floor of mouth Do not confuse with mucocele (mucous extravasion phenomenon) nodule of saliva due to escape from duct of salivary gland Which gland (parotid, submandibular, or sublngual) is a sialoth most common Which space is not involved in ludwigs angina? (submental submandibular, retropharyngeal, or submental) Impression materials Irreversible hydrocolloid (alginate) rapid set, pour immediately, not accurate (dx casts only) Reversible hydrocolloid (agar) hydrophilic, long working time, accurate, pour immediately; no custom tray but requires special equipment, tears easily Polysulfide polymer (for complete dentures) high tear strength, smells, messy, good reproducibility, pour within 45 minutes; H2O is a byproduct contraction Condensation silicone (?) short setting time, pour immediately; EtOH is a byproduct dimensional contraction; base+acc Addition silicone (VPS) no byproducts dimensional stability, low tear strength, fewer voids when poured immediately Polyether (Impregum) base and accelerator; no byproducts excellent dimensional stability; pour soon Malleability deform (without fracture) under compressive strength; ability to form a thin sheet; gold is malleable Greatest malleability to least: gold, silver, lead, copper, aluminium, tin, platinum, zinc, iron, and nickel Ductilty deform (without fracture) under tensile strength; ability to stretch into wire greatest ductility to least: gold, silver, platinum, iron,nickel, copper, aluminium, zinc, tin, and lead. Gold inlay/onlay divergent walls (2-5 degrees per wall), 30 degree bevel margins for better fit, skirt extend beyond line angle RPD classifications class I bilateral edentulous area posterior to remaining teeth class II unilateral edentulous area posterior to remaining teeth class III unilateral edentulous area with natural teeth anterior and posterior class IV single bilateral edentulous area with natural teeth remaining on both sides RPD components Major connector connect all rpd components on one side to those on other side Minor connector stabilization Rest support Clasp arms stabilization (middle 1/3) and retention (gingival 1/3)

Epidemiology definitions Incidence - measure of the risk of developing some new condition within a specified period of time Prevelance - measure of the total number of cases of disease in a population Sensitivy - measures the proportion of actual positives which are correctly identified as such Specificity - proportion of negatives which are correctly identified (e.g. the percentage of healthy people who are correctly identified as not having the condition). Pictures

Dentinogenesis Imperfecta Lymphoid epithelial cyst odontogenic origin Cemetoblastoma RO mass that replaces root; tooth is removed with lesion Periapical Cemento-Osseous dysplasia vital, lower anteriors, middle age women, RL then RO; no symptoms Florid Osseous dysplasia involves entire jaw Migratory glossitis Nicotinic Stomatitis Questions Pt with inlay has pain during biting..no radiographic evidence m-d cusp fracture poor prognosis o M-D fracture cant be seen radiographically because the fracture line is not in the plane of occlusion Cracked tooth with more symptom to---cold,heat,pressure (hard to reproduce), sweets, acidic foods also o Commonly lower first molar o Dx: stain, transillumination, tooth slooth, air to detect where pain from crack occurs o Tx healthy/reversible splint/observe or crown (temporary then permanent); irreversible/necrosis endo, core (no post if possible, crown Most common area of fracture in children: symphysis, condyle, coronoid Apical root closes in about 3 years from eruption o Eruption occurs through gingiva with root completion o From calcification to root completion: 10 years (canine 13 years) Tooth with most favorable prognosis---small internal resorption Esthetic analysis, face divided: o Vertically by 5 (4 planes) o Horizontally - 3 (2 planes) o Frankfort horizontal line - porion to orbitale o Campers line - occlusal rim parallel to ala tragus line Sterilization must kill spores of spore-forming bacteria (Bacillus and Clostridium) o Bacillus spores are the benchmark organism for sterilization o most destructive to carbide instruments----steam heat 250 degrees for 15-20 minutes; denaturation o dry heat does not Corrode or dull instruments; only glass and metal; 320 degrees for 1-2 hrs o cold sterilization soak in chemical then rinse in sterile water; 10 hrs to kill spores with 2% glutaraldehyde Disinfection antimicrobial kills or stops growth of pathogens; applies to inanimate objects o Mycobacterium tuberculosis is the the benchmark organism for disinfectants; spores are not killed o Something about disinfecting a surface, I think you leave it for 10 minutes then wipe Antisepsis applied to living tissue; alcohol Orange stain is important ---to change chroma o Most common staining on porcelain

Which endodontic procedure is most unsuccessful in primary tooth with deep caries? Direct pulp capping may cause internal root resorption Resorption of bone takes place in which direction after extraction----downward inward o Other choices: downward outward, forward inward Which procedure least likely to produce bacteremia--extraction, non surgical endo, oral prophylaxis Which gracey curet is used for the mesial surface of distal root in max tooth---11-12, 13-14 Which determine energy level of photon in xray---kvp,ma Units o o o o

Exposure Roentgen Absorbed dose Gy Effective dose Sv Radioactivity Bq

Which structure is most radiosensitve----hemopoitic bone marrow , also oral mucous membrane o Cells that are mitotically active and undifferentiated and have long mitotic futures Reversal occlusal plane---chin tilted too upward Which is most important for diagnosis of maxillary sinus xray----occlusal,panaromic,MRI Effect of xray---genetic mutations Collimation reduce SIZE of x-ray beam reduce volume of irradiated patient tissue o Circle 7cm in diameter Tranillumination method is most useful in carious diagnosis ---anterior proximal, posterior proxmal 15% Aluminium chloride isHEMODENT most commonly used; contains no epinephrine o USE OF ZINC CHLORIDE IS NOT RECOMMENDED necrosis o Epinephrine is not used due risk of increasing BP in hypertensive pts o Why do you keep sulcus dry when placing cord? Porcelein laminate veneer fixed by resin show black margin in 2 days cause---porcelein break,resin wears off??? Amine?as Ultrasonic Instruments: active portion is the tip, 20-45k cycles/seconds o Magnetostrictive elliptical vibration pattern, all sides of tip are active o Piezoelectric linear vibration pattern, 2 sides are more active o CONTRAINDICATED in patients with Pacemaker, communicable diseases, titanium implants (use plastic tip) o Something about what determines the effectiveness of the ultrasonic instrument. Something about the tip Troches available fungal agent---clotimazole for treating oral Candidiasis; also Nystatin Which is not an antifungal? Options were ketoconazole, Nystatin, Clotrimazole, and something else U shaped radiolucency in max molar---zygomatic process J shape radiolucency - vertical root fracture Antral Y Pt says I dont have time to stop smoking contemplatory, precontemplatory, denial,action Elderly people abuse question --under reported Adult child can decide a treatment plan of his old patient his parent in what suitation---when he is payig money,when he is impaired either ear or eye,patient want his decision has power of attorny

Dentist does the treatment for 2 crowns but the insurance company pays the money for one crown downcoding. o The alteration by an insurer or other third-party payer of service codes for physicians or other health care providers, to those of lesser complexity, resulting in decreased reimbursement o Q was either about downcoding or upcoding, know both which area is most difficult to achieve matrix adaptation max mesial 1 prem also common place for perforation Which impression material has good wettability---Hydrocolloid > Polyether > Hydrophilic Addition Sillicone > Polysulfide > o Hydrophobic addition silicone and condensation silicone has same wettability uses of chlorhexide--reduce plaque accumulation Epidemiological studies o Cross sectional: Sample of people assessed at one time o Case control: people with (cases) are compared to people without (control), but are otherwise similar May establish exposure-disease relationship o Prospective cohort: follow group over time; describe prevalence of outcomes in the group o Retrospective cohort: choose a group that HAD the outcome of interest evaluate effect that an exposure had on a population (ex occupational hazards) o Double blind clinical trial: neither subject nor investigator knows which group is control or case Used to compare incidence and side effects o Community trial: community as a whole is studied; the more similar the communities, the more valid the results Ex. Fluoridation trial of 1945 (fluoridated community vs non-fluoridated) Pt has composite restoration with severe pain with localized swelling---- Incision & Drainage Action of sodium hypochlorite disinfection of root canals, dissolves organic matter; DOES NOT REMOVE SMEAR LAYER Most crucial in replantation----time managment Ectopic eruption of man 1st molar in relation to pri man 2nd molar cause some resorption -management---extraction of 2nd molar,separation, disking of 2nd molar Calcification of premolar tooth at birth----NO Patient with 12 primary and 12 permanent teeth teeth---81/2 years old o Primary: U/L 3,4,5s; Permanent U/L 1,2,6s o Eruption of what teeth at 11 years old Mouth breathers have facial feature of incompetent lips, convex profile, narrow palatal vault, bilateral cross bite Most common cause of frequent urination during 3 trimestor--pressure of uterus on bladder,gestatory diabetic Facial profile of class 2 malocclusion--convex, Class III is concave

Normal biological width 2mm (0.97 mm JE, 1.07 mm CTA) + sulcus o Biologic width is the distance established by "the junctional epithelium and connective tissue attachment to the root surface" of a tooth. This distance is important to consider when fabricating dental restorations, because they must respect the natural architecture of the gingival attachment if harmful consequences are to be avoided. Acid etching show decalcification at the area but did give the appearance of frosty why more fluoride in tooth, therefore you have to etch longer or again. MUST HAVE FROSTY APPEARANCE Beveling in acid etching composite use more surface area. Exposes the enamel rods obliquely o Something about which is not a reason to bevel; the answer will be obvious Distobuccal flange of denture determined by---masseter Removing the mylohoid ridge have the common error in--lingual nerve damage Cheek bit due to---horizontal overlap (edge-to-edge), or too far buccal or lingual if it is mandibular facial placement of teeth, then it is the option. Ant flange in max teeth---esthetics Facebow transfer for recording---max with hinge axis positioning Post op managment after denture delivery pt has pain in crest of the ridge---defect in post occlusion (or whatever area) o Excessive VD, bubble in acrylic, inaccurate denture base, Placement of rubber dam affect the colour selection by dehydration of tooth gives inaccurate tooth shade Know about the tooth reduction for various materials Tooth set in 20 degree for balanced occlusion...but adjusted to 45 degree what is need to corrected for balanced occlusion-compensatory curve DNA probing in perio pocket --for bacterial specific Tooth with endo treated and post with crown have pain after several days esp during biting and cold..vertical root fracture Use of indium with alloy is mainly to provide chemical bond with porcelain Classification of pontic mainly depend on their relationship ---to the ridge (mucosal or nonmucosal) o Mucosal: concave and passively contact the ridge Saddle Ridge lap NOT USED because it forms a concave area B-L that is uncleasable Modified ridge lap very esthetic, moderately easy to clean, for PFM, all-metal, or all-ceramic Ovate very high esthetics, good for high smile line, easy to clean, requires surgical preparation Conical Molars without esthetic requirement, easy to clean Bullet shaped o Nonmucosal used in nonesthetic areas Sanitary (hygienic) about 2mm from mucosa, all metal pontic Modified sanitary (hygienic) Know about pier abutment and cantilever o Cantilever teeth with RCT are not preferred for cantilever abutment; should have more than sufficient bone support o Pier Abutment single tooth with two edentulous spaces on either side. In this case the single tooth will have to act as an abutment for both the edentulous spaces in the bridge. In order to prevent trauma to the abutment a stress breaker is provided with in this sort of Dental Bridges near the pier abutment. The stress breaker is a nonrigid connector with a key in key-way. If the pier abutment is mobile then a rigid connector should be used instead of non-rigid connector.

Q about which abutment/cantilever system will have the most traumatic effect on the abutment. Choices were molar abutment, premolar pontic, max lateral abutment, max central pontic, max central abutment, max lateral pontic, and max canine abutment, lateral pontic. (I think)

Physiological rest position Why do the patients teeth click when they wear dentures Working side and non working side interfence Bennett shift mainly on --lateral movement of condyle or working side o Aka Immediate side shift The bodily lateral movement of the mandible towards the working side during lateral excursions (approx. 0.3mm) Narcotics mainly contraindicated in ---MOA inhibitor Severe alcoholic now recovering need 24 tooth extraction which test needed----INR CBC Property of interocclusal recording material----low resistance to jaw closure What is the disadvantage of using elastomer for jaw relations or CR (something like that). Like what will happen when you try to mount? Incisal guidance mainly depend on ---hori and verti overlap Occlusal trauma symptoms - fremitus is the best test to test TFO, vascular - blood clots in PDL, gingival recession, widening of PDL Erythema in the palate of the patient wearing denture...inflammatory papillary hyperplasia o Poor hygiene, ill-fitting dentures, denture overuse After having denture (partial) the pt have pressure for few days, later it subsides---occlusal trauma, pressure of clasp, pulp damage Which doesnt need replacement or repair--recurrent carious at margin,pulpoaxial line fracture, ditched restoration Pt has restoration shows demarcation b/w restoration, which is fracture in middle of restoration. Whats the next step--look for further fracture line, replacement of restoration...? Sensitivity following composite restoration in post most comman cause---due to resin,polymerization shrinkage in margin,shrinkage floor...??? Without indirect retainer---outward displacement of distal extension base Flexiblity of alloy depend on all expect----composition, taper, undercut (ans) Epithilium of free ging graft----degenerate Distractive osteogenesis vs. osteotomy o distractive osteogenesis is a surgical process used to reconstruct skeletal deformities and lengthen the long bones of the body. o An osteotomy is a surgical operation whereby a bone is cut to shorten, lengthen, or change its alignment Bacteria in healthy mouth---facultative gram positive anerobic bacteria

Which of the following is not true about local agressive periodontitis----affect less than 30%, treatment scaling and systemic antibiotics, genetic Disease with Desquamative gingivitis o lichen planus, mucous membrane pemphigoid, and pemphigus A band of red atrophic or eroded mucosa affecting the attached gingiva is known as dequamative gingivitis. Unlike plaque-induced inflammation it is a dusky red colour and extends beyond the marginal gingiva, often to the full width of the attached gingiva and sometimes onto the alveolar mucosa Lateral perio cyst common location---bicuspid lower Primary reason for replacing an overhang restoration---interfere in plaque removal Which one is common in pregnancy and in normal condition--pyogenic granuloma Best longterm care after perio treatment---self,professional..??? Which type of interleukin in most common after perio diseaseIL-1 If implant with width of 4mm is used what should be the bucolingual width of the ridge? 6mm o minimum Vertical height of bone to place implant - 8mm o minimum Width of bone is 6mm choices were 5mm, 7mm, 8mm, 12 o minimum distance of apex of implant From nerve - 2mm o platform of implant from adjacent CEJ - 2-3 mm o between implants 3mm o between implant and tooth (height of coutour) is 1mm o Mini implant is 2.4mm o When there is FPD from natural tooth to implant, the max stress is concentrated on the SUPERIOR PORTION OF THE IMPLANT RPD: o o o

Retention: Direct retainer , and indirect retainer Stability: Minor connectors, proximal, ligualul plate, denture base Support: Rest, major connectors, denture base

H2O2 - less than 10% OTC o H2O2 - 30% (superoxol) used in in-office bleaching o Carbamide peroxide - 15-18% tray bleaching (commercially available in syringes) Most acceptable root sensitive theory---hydrodynamic o the flow of fluid in dentinal tubules trigger receptors within the tooth Attrition---normal wear Re-evalution after perio therapy---4-6wks

Pt with diabetes having sedation IV and LA---ask the pt to take high calorie food with insulin, low calorie food which inusulinno restrictions Mucosal graft epithelization by---connec tissue from underlying tissue(recipient site)

Lefort I associated with- what fracture--nasoethmoidal air cell, frontal sinus, max sinus, mastoid air cell o Le Fort II - separation of the maxilla, attached nasal complex from the orbital and zygomatic fractures o Le Fort III - Nasoethmoidal complex, the zygomas, and the maxilla from the cranial base which results in craniofacial separation Treatment of mid face deficiency is 1) lefort I 2) lefort II 3) lefort III Apexogenesis vs. Apexification (know when to do either) o Apexogenesis: vital pulp therapy for immature tooth with incomplete root, damaged coronal pulp but health radicular pulp Allows continued development of the entire root Encourages body to make a stronger root and eventually root closure o Apexification: nonvital tooth Stimulates formation of calcified tissue at the open apex of pulpless teeth Create proper environment for formation of the calcified barrier by cleaning and removing debris/pulp and place material (calcium hydroxide) to induce apical closure Symptoms of Sjogren immune cells attack and destroy exocrine glands o Sicca symptoms: Xerostomia, xerophthalmia, general dryness, arthritis, lymphoma Acute gingivostomatis virus associate with---chicken pox virus also Acylovir---in some viral infection

Most common cause of failure of in restoration in primary tooth---cavity prep,moisture contamination... Fusion---two tooth join only by dentin While giving inf inferior alveolar injection the pathway from?contra lateral premol,contra canine,contra molar,ipisi lat premol How to split the tooth using bur tech of tooth removal---spilt buccal and lingual up to furcation Constantly exposing the pt to get from the fear factor is---desensitation

Fear decreases pain and anxiety increases pain Most common site of herpes--attached gingiva Scopolamine commonly used for motion sickness o Anticholenergic drug he drug is used in eye drops to induce mydriasis (pupillary dilation) Prostaglandin inhibitor cause all except---increase gastric mucous. o PG:decrease gastric acid and increase gastric mucous, inc blood flow in kidneys Unconsicous most commonly---psychogenic Asthmatic patient, NSAID contraindications: NSAIDS cause bronchospasm. longterm asthma give corticosteroid Pt with osteoradionecrosis often have 4-5gy of radiation therapy Pt with bisphosphanate 3ml IV for 3 yrs, have carious and unrestorable tooth; management: stop regmen 1 mnth extract, do non invasive endo treatmnt. o bisphosphonate has half life of 6 months, it is advisable to wait for atleast 6 months before invasive procedures Most common side effect of N2O2---nausea and vomitting Laryngeal obstruction in---anaphylatic shock

Steriod supplement indicated in pt with---10mg with 1 yr, 10 in 2yr Erosive crust in skin with target----erythema multiforme Diazepam action in GABA both relaxing, anti-anxiety and anti-convulsive effects Common malignant potential---Paget's o Maxilla>mandible, o Lion-like facial deformity. o Teeth demonstrate extensive hypercementosis. o Development of malignant bone tumor usually an osteosarcoma is recognized complication of Paget, frequency of osteosarcoma is 1%, mostly long bone Absence of primary tooth---premant eruption slow Visible light curing - 400 - 499nm Advantage of gold on occlusal surface, porcelain in facial surface----conserve tooth struc,minimal rsduction...???? Why do you bevel the functional cusp for a full coverage crown? Interaction of proponolol (B-blocker) with epinephrine is best described by synergism o Increases the effects of epi so risk for Hypertension and bradycardia o Better to use a selective B1 blocker such as metoprolol Drug most likely cause xerostomia---lithium (antipsychotic), ca channel blockers (like nefidipine), and anticonvulsants. Pt have swelling after extraction, he is under penicillin therapy. What would be the next drug of choice? Clindamycin Beta lactamase---inhibit the action of penicillin o Methacillin (rarely used), Flucloxacillin, Oxacillin, Dicloxacillin are resistant to beta latamase Both drug have same intrinsic effect and different receptor affinity---same potency, same efficacy Pt is addicted to oxycodone (opiod) which contraindicated codiene(opiod), pentozocaine Antipsychotic with irreversible side effect----Tardive dyskinesia ---Sudden purposesless movement(parkinson like disease) Lantaprost indication----Glaucoma(Latanoprost) Large radioopaque lesion in carious affected tooth---condensing ostitis Pierre Robin Syndrome: o glossoptosis, retrognathia, upper airway obstruction, and cleft lip or palate o Child will have difficulty breathing and feeding Treacher-Collins Syndrome o downward slanting eyes o micrognathia (a small lower jaw) o conductive hearing loss o underdeveloped zygoma o drooping part of the lateral lower eyelids o malformed or absent ears. Granulomatous lesion---crohn's disease o Also Wegeners syndrome, TB, Histoplasmosis, Sarcoidosis, Cat -scratch disease

Patient with labial lesion for 7 days, similar kind for lesion wk before---recurrent apthous Supparative lesion---actinomycosis o Actinomycoses isrealii is normally found in nose and throat Symptoms occur when the bacteria enters the facial tissues after trauma, surgery, or infection. o A common triggering causes is dental abscess or oral surgery o Once in the tissue, it forms an abscess, producing a hard, red to reddish-purple lump, often on the jaw, from which comes the condition's common name, "lumpy jaw." o Eventually, the abscess breaks through the skin surface to produce a draining sinus tract. o Find Actinomyces and sulfur granules in drained fluid Hypercemtosis o Etiology of Hypercementosis can be caused by many things. "PIG ON TAP": Pagets, Ideopathic, Gigantism Occusal Trauma, Non functional tooth Trauma, Acromeglia, Periapical granuloma Localization of mesiodens is done with occlusal radiograph Neuropraxia: It is a transient episode of motor paralysis with little or no sensory or autonomic dysfunction o Neurapraxia describes nerve damage in which there is no disruption of the nerve or its sheath St John's wort is most widely known as an herbal treatment for depression (300mg of extract) o may work against gram negative bacteria, ADHD Pt is a child and is diabetic undergoes hypoglycemia in the chair if conscious give him orange juice o If unconscious give him 50% dextrose IV o Type I Diabetes leads to a) Aphasia b) Ataxia c) Blindness d) Deafness Amantidine is used as Antiviral for Influenza A o Also for Parkinsons (may promote dopamind release in substantia nigra) Strawberry tongue seen in scarlet fever o Also in Kawasaki disease and toxic shock syndrome Prophylactic treatment o for pacemaker no premedication required o for Prosthetics heart valves premedication required LA calculations 4.4mg/kg o One carpule of 2% xylocaine has 36mg Primate spaces are seen between maxillary Lateral incisors and canines; mandibular canines and molar Which of the following Medication is not by cell wall synthesis? o Penicillin B) Amoxicillin 3)Vancomycin 4)Azithromycin - inhibits protein synthesis o Protein Synthesis CLEAN(50s):Clindamycin, lincomycin, erythromycin, azithromycin, chloramphenicol, clarithromycin TAGS(30s): tetracycline, aminoglycosides, gentamycin, streptomycin Questions on public health case studies o Prospective cohort studies: A prospective cohort study is a research effort that follows over time groups of individuals who are similar in some respects (e.g., all are working adults) but differ on certain other characteristics (e.g., some smoke and others do not) and compares them for a particular outcome (e.g., lung cancer).[1] It should be emphasized that prospective studies begin with a sample whose members are free of the disease or disorder under study (e.g., free of lung cancer or free of major depression). o Retrospective cohort studies: A retrospective cohort study, also called a historic cohort study, is a medical research study in which the medical records of groups of individuals who are alike in many ways but differ by a

certain characteristic (for example, female nurses who smoke and those who do not smoke) are compared for a particular outcome (such as lung cancer). A retrospective (historic) cohort study is different from a prospective cohort study in the manner in which it is conducted. In case of Retrospective Cohort Study, the investigator basically collects data from past records and does not follow patients up as is the case with a prospective study. However, the starting point of this study is the same as for all Cohort studies. The first objective is still to establish two groups Exposed versus Nonexposed; and these groups are followed up in the ensuing time period. Case-control is a type of epidemiological study design. Case-control studies are used to identify factors that may contribute to a medical condition by comparing subjects who have that condition (the 'cases') with patients who do not have the condition but are otherwise similar (the 'controls').

What liquids used in Glass Ionomer Cement Polyacrylic Acid o In most of the current cements, the acid is in the form of co-polymer with itaconic, maleic or tricarboxylic acids. These acids tend to increase the reactivity of the liquid, decrease the viscosity and reduce the tendency for gelation Radiology o Focal spot influences resolution o Collimation influences penetration o Milliampere influences intensity o KVP influences energy and pentration A pt is taking methotrexate (immunosuppressant) will have drug interaction with o Beta blockers B) Alpha blockers C) NSAIDs D) Beta lactamase o *Never use with Acitretin or Asparaginase (absolutely contraindicated!) o With methorexate we cannot give amoxillin because it decreases renal clearance Most common o Odontogenic cyst is periapical cyst (non-vital tooth) o odotogenic ectodermal is ameloblastoma o odontogenic mesenchymal is fibroma o non-odontogenic cyst of the oral cavity is the nasopalatine duct cyst o salivary gland tumor is pleomorphic adenoma Recommended daily intake of fat Intake as per USDA is 30% of total daily caloric intake or 10% saturated fat intake of caloric intake American Society of Anesthesiology patient classification status o ASA I: Normal healthy Pt o ASA II: No functional limitations; has a well-controlled disease of one body system; controlled hypertension or diabetes without systemic effects, cigarette smoking without chronic obstructive pulmonary disease (COPD); mild obesity, pregnancy o ASA III : Some functional limitation; has a controlled disease of more than one body system or one major system; no immediate danger of death; controlled congestive heart failure (CHF), stable angina, old heart attack, poorly controlled hypertension, morbid obesity, chronic renal failure; bronchospastic disease with intermittent symptoms o ASA IV : Has at least one severe disease that is poorly controlled or at end stage; possible risk of death; unstable angina, symptomatic COPD, symptomatic CHF, hepatorenal failure o ASA V : Unstable moribund Pt who is not expected to survive 24 hours with or without the operation o ASA VI : Brain-dead Pt whose organs are removed for donation to another

Periapical cyst Odontoma Recurrent apthous ulcer Thyroglossal cyst

Location of Inferior Alveolar Nerve - They then pierce the buccinator muscle between the palatoglossal & palatopharangeal folds, lying lateral to the medial pterygoid at the mandibular foramen. Syphilis Chancre resembles 1) Cancer 2) Herpes 3)Herpangina 4) Apthous Ulcer

Odontogenic Myxoma: o Most common odontogenic tumor of mesenchymal origin o Post Mand o Honeycomb & multilocular appearance o Rx: similar to ameloblastoma & Giant cell Granuloma o Tx: Curettage, possible recurrence

Ameloblastoma: o Most common EPITHELIAL ODONTOGENIC TUMORmand molar area o Age 40s 50s o Histo: reverse polarity Ameloblastic fibroma: compared to ameloblastoma o younger age o slower growth o does not infiltrate Ameloblastic fibro-odontoma o similar to the above except it occurs in the MAXILLA AND MANDIBLE in equal freq Ameloblastic odontoma: o same as above except it occurs in max & mand pre-molar & molar area All of the following applies to oral cancer except o 1)Male 2) Smoking 3)African American 4)Low socio economy 5)sex predilection for location Which of the following will not occur in over contouring of crown is o A)Gingival problems 2)caries on the adjacent tooth 3) bone loss Which of the following will incur more force on opposing dentition? o complete denture b) tooth borne partial c) tissue born partials d) over dentures enamel pearls most commonly seen on maxillary Molars gingival palatal groove most like seen on seen in Max laterals Anatomy on maxillary central that aids in plaque retention cingulum

Radiology o o

mean energy of Xray photons is increased by increasing KVP collimation makes the X ray photon decrease the radiation to the pt Fogging of film is in overdevelopment, contaminated sols, deteriorated films and light leaks more Kvp - less contrast for restorative purposes Dark radiographs - Overdevelopment, excessive mA, excessive peak kilovoltage, film-source distance too short.

Osteoradionecrosis occurs because of decrease in vascular supply o Traditionally 1-2 wks btw TE and radiotherapy is suggested. however is better to delay radiotherapy 3 wks after TE. o After Radiotherapy if tooth has necrotic pulp o endodontic Tx with systemic antibiotics can be performed o if difficult to do RCT (because of sclerotic pulp)>>>tooth can be amputated above the gingiva and left in place Pt is on IV bisphosphonates which of the following procedures can be done o 1)Scaling and Prophy 2)endodontics 3) surgery 4) extractions Crouzon's syndrome exhibits severe proptosis (exapthalmous) o seen in Crouzon's syndrome: hypertelorism, underdeveloped maxilla, brachycephaly Which endocrine system does thick hair become thin hair thyroid - hypothyroidism o (cretinism in kids and myxoedema in adults) Extrusion of canine what flap technique is used except 1)Envelope flap 2) Semilunar flap 3) Apical repositioning flap Know about flaps (incisions etc) Collagen disorder seen in advanced Diabetes and Rheumatoid Arthritis Mechanism of action of substances on GABA receptors o increasing the frequency of chloride channels by Benzodiazepines o Barbiturates increase the duration of chloride channel opening Neurotransmitter in Parkinsons disease is Dopamine Pt has Asthma and is allergic to Aspirin what pain medicaction will be given? o Acetamenophen 2) Ibuprofen 3) Diclofenac Sodium o NSAIDS - are contraindicated in asthma pts. When you place a implant, widening of crestal bone is seen because of which force? o Horizontal 2) Oblique 3) Vertical 4) Aapical o Fibers on implant What is the reason for unhappiness of denture Instability of the lower denture Mode of action of miconazole - alters the cellular wall permeability. o It works by inhibiting the synthesis of ergosterol, a critical component of fungal cell membranes.

Cause of indiscrete margins on xray film Antibiotic contraindicated with ALCOHOL are Metronidazole, Tinidazole, Antimalarial, flurazolione, Griseofulvin o headache, nausea, vomiting, irregular heart beat, tachycardia, flushing, breathlessness, low BP In a # of rt side of body of mandible, which other # is to be suspected - condyle on the contralateral side of the subcondylar region o Trauma to one side often produces an ipsilateral body fracture and a contralateral subcondylar fracture. o A heavy blow to the symphysis produces a symphyseal fracture and bilateral subcondylar fractures. o It is also important to exclude damage to the cervical spine and to ascertain that the airway is not compromised. In condylar hyperplasia, mandible deviates to which side? to the contralateral side (the unaffected side), in condylar hypoplasia the mandible deviates to the affected side Treatment of ranula (floor of mouth): a) marsupialisation b) enucleation c) inj steroid; Treatment of a mucocele enucleation

Pt complains of high fees of dentist, how should the dentist answer? Fee is fine according to the geographic area, it is fair and reasonable, I have to make a living too Handling of an ADHD pt and how to get them to maintain oral hygiene - no modifications Working side interferences? Max: palatal inclines of buccal cusps, Mand: buccal incline of lingual cusps o In MIP or CO, Max: buccal incline of palatal cusp, Mand: lingual incline of buccal cusp o Balanced side interferences are Max: buccal incline of palatal cusps and Mand: lingual incline of buccal cusps Protection for inf alv nerve while placing an implant : surgically move the nerve, meticulous placement of the implant, place a barrier Action of chlorhexidine? membrane disruption o Antiseptic and antimicrobial with bacteriocidal activity Denaturation of the proteins - alcohol and autoclave; Coagulation of proteins - dry heat Source of epithelium for graft: host, graft, adjacent tissue. Source of the blood supply is the host connective tissue Disadvantage of hydroxyapatite graft - control of granules is hard and in the region of lower bicuspds, graft might be solidify over the mental foramen and cause parasthesia. After flap surgery, how does repair occur? Pdl moves occlusally, apically, laterally

Upon using the fluorescent light, in which of the conditions is the complete tooth illuminated: vertical fracture, periapical abscess, split tooth? Color of nitrous oxide cylinder - blue o oxygen cylinder is green Which does not describe a biohazard waste box: made of metal, closed, puncture proof, leak proof Some law on hazard communication law. What is it and what organization created? Pt swallows a crown, what position should you place them in For a pt with head and neck cancer who is to be irradiated, when should be questionable teeth removed: before irradiation, after, during? Treatment for lingually erupted #9 at age 7 no tx Mandibular incisors erupt in what position to the primary teeth? Buccal to, lingual to, straight under

Advantage of bundling instruments before sterilization precleaning in ultrasonic? What happens if penicillin and erythromycin are given together: summation, potentiation, idiosyncrasy Tetracycline cancels out Penicillin Numbers of surgical forceps for tooth extraction 150 = upper, 151 = lower; What forceps would you use to extract mandibular premolar? Choices were like 150, 151, 23, 71 Angulation of cutting edge to the tooth surface while root planning more than 45 less than 90 After using a gingival retraction cord, tissue reacts by recession. Where do you see this the most; lingual, buccal, interproximal. Which aspect most commonly needs convincing for the pt with new CD: speech, chewing, max denture retention, mand denture retention Which of the following is absent in facial palsy: drooling, inability to wink, loss of muscle tone, excessive salivation o May be caused by Herpes Zoster, Sarcoidosis, middle ear infection, Lyme disease, HIV infection o May also experience: dry mouth and dry eye, headache, loss of taste, hearing louder in one ear

Consequences of tooth ext in a pt with adrenaline crisis probably delayed healing, infection, inflammation, LoC, dizziness, tachycardia, tachybradia o If Pt uses more than 10 mg prednisone daily then: double the dose of daily steroids at the day of surgery as well as double the dose the day after surgery. (always after consultation with MD Contraindications for the use of opiates cannot be used with MOA inhibitors, incase of head injury Bacteria that is responsible for progression of caries but not causing the initial phase - Lactobacillus Patient presents with eroded occlusal surface, lingual surface of max teeth, fillings raised from the surface. What should be suspected possible GI regurgitation Radiographic feature of osteoporosis o Osteoporosis results in an overall reduction in the density of bone. o This reduction may be observed in the jaws by using the unaltered density of teeth as a comparison. o There may be evidence of a reduced density and thinning of cortical boundaries such as the inferior mandibular cortex The caries rate in patients with Downs Syndrome is less. o The patients with DS had a significantly lower prevalence of caries and significantly higher levels of salivary sIgA in this study. This finding tends to support the hypothesis that higher levels of salivary sIgA may protect against dental caries. o But periodontal disease is more Cleft lip and palate is usually associated with which kind of malocclusion? class III Purpose of dietary analysis of pt -Caries risk assessment

Cause of dry socket (alveolar osteitis) : fibrinolysis of clot, physical dislodgement of clot? o How do you treat it? (make sure you recognize dry socket as alveolar osteitis o Multiple types of alveolar osteitis can result from disturbances in the healing process. The type that is commonly referred to as dry socket is one in which the disturbance is from the time a blood clot forms immediately after tooth extraction to the initiation of healing in the 4-5 day period after extraction occurs. o The healing tissue that is supposed to replace the blood clot, known as granulation tissue, may fail to grow or be disrupted after beginning to grow, leading to the well known symptoms of dry socket. Which of the following is a dentist not supposed to do: talk about options provided by other specialist, risks of not having tx done, benefits of having treatment done, discuss about copay Note:If you see a fact e.g broken file,or tooth perforation you have to inform the Pt but not blame the previous dentist Onset of action of antipsychotic is 5-6 days Child with blue lips, thickened nail beds, pale skin. What disease is to be suspected? Congenital Cyanotic heart disease Muscle that decides posterior extension of lingual side of dental flange? Mylohyoid, superior pharyngeal constrictor? Define chief complaint - it should be in pt's own words Oral hygiene instructions for ADHD pt: hygienist to pt, hygienist to pt and parents, dentist to pt, printed material

All of the following can be used for plaque removal except: water irrigation, tooth picks, toothbrushing, floss What is water irrigation used for? Tooth brushing technique best suited for removal of interproximal plaque-Modified Charters Where does interproximal caries generally occur: above contact area, below contact area, at contact area, area between marginal ridge and contact area? What does wheezing sound like? o gasp followed by high-pitched whistling Pregnant woman goes into syncope in second trimester. What should be the immediate action? o turn her to the left in order to remove the pressure from inf. vena cava Epidemiology of oral disease o Diabetes is common on which race? black men o Periodontitis is common for which race? Black o What group has the highest prevalence of coronal caries? Blacks, Whites, Native Americans , Hispanics. Prevalence of caries in permanent teeth (DMFT) adults 20 to 64 years of age, White >Black>Hispanic Prevalence of untreated decay in permanent teeth.. Black >Hispanic> White Mean number of decayed, filled, and decayed or filled permanent teeth among adults 20 to 64 years of age White>Hispanic>Black Prevalence of Class II more in WHITES, Class III in Blacks Class III malocclusion with cleft lip and palate is more in Native Americans > Oriental, Caucasians > Blacks. Cleft lip is more common in Asians Use of pulp testing: check disease or health or extension of damage of pulp, check the responsiveness of pulp nerves. How does a tooth covered with crown react to pulp testing--- cold is better test Night guard is used for: treating bruxism, redistribute occlusal forces Should be able to identify defect in hue, color, chroma on a photograph. Displacement of fractured segments of mandinble by the action of associated muscles. If the # is at symphysis the muscles are genioglossus and anterior digastric. Pt is addicted to oxycodone which contra indi- Patients receiving CNS depressants such as other opioid analgesics, general anesthetics, phenothiazines, other tranquilizers, centrally-acting anti-emetics, sedative-hypnotics or other CNS depressants (including alcohol) concomitantly with PERCOCET tablets may exhibit an additive CNS depression. o When such combined therapy is contemplated, the dose of one or both agents should be reduced. o The concurrent use of anticholinergics with opioids may produce paralytic ileus. Bioavailability---amount of drug avilable in systemic circulation Question about effect of a drug that has high first-pass metabolism o This first pass through the liver thus greatly reduces the bioavailability of the drug Antipsychotic with irreversible side effect----tardive dyskinesia (2Q) Lantaprost indication---- Xalatan (latanoprost ophthalmic) eyedrops for glaucoma o Reduces intraocular pressure o Prostaglandin analogue

Common site for granular cell myoblastoma (granular cell tumor)---tongue Large radioopaque lesion in carious affected tooth---condensing osteitis Lesion commonly with dysplasia and carcinoma in situ---erythroplakia Common finding in patient with ataxic cerebral palsy---ant tooth fracture

Common reason for angular chelitis---decreased vertical dimension (in elderly), skin fold, poor diet (Vit B2 and B12) o Seen in pts with Anorexia/bulemia o In edentulous pts, when there is decrease vertical dimension, most common sign is ANGULAR CHELITIS o Can be fungal (Candida) or bacterial Ideal depth of a bone for an implant 8mm Clift lip prevalence in USA is 1:700- 1:800 (other source says 1:1000); occurs week 4-5 in utero o Cleft lip is done in 2-3 months after the child is born o The lip is corrected as early as is medically possible. The soft palate is closed btw 18-24 months of age

Clept palate occurs weeks 6-8 in utero. The hard palate cleft is then closed around age 4 or 5. o Alveolar correction surgery is at age 8 before the eruption of canine o Correcting the alveolar cleft is usually left till a later time, around the age of 8 or 9. If the surgery is conducted when the child is younger than this, the scar tissue that forms during healing will interfere with the normal development of the face. o An alveolar cleft is corrected with a graft of bone and soft tissue Endo o o o

o o o

Sodium hypochlorite in root canal treatment has the following advantages EXCEPT: 1.lubricating, 2.Anti-microbial, 3.Chelating agent, 4.Disinfection Gutta percha has the following advantages EXCEPT: 1.easy manipulation, 2. Adapts to tooth surface, 3.Anti-microbial 4. Biocompatible During root canal treatment, the operator creates a ledge. What is the next step? 1. Fill up to the ledge and leave the ledge intact, 2. Extract and replace with an immediate implant, 3. Take a smaller file to working length, remove as much debris as possible and gently try to remove the ledge. What is the NOT an advantage of stainless steel files? 1. More flexible, 2. Less chance for breaking, 3. Allows the file to be centered in canal Before separation, SS files exhibit fluting distortions, highlighting unwound or twisted regions of the file signs of fatigue Nickel-titanium files do not show these same visual signs of fatigue. Discard before visual signs are seen Primary endo infection STRICT ANAEROBES: G- Porphyromonas, G- Bacteriodes, G+ Actinomyces (root caries) Unsuccessful rct need re-tx from persistent infection of facultative anaerobes Abts: DOC: Pen VK good for strict and G+ facultative anaerobes Metronidazole is effective against strict anaerobes but not facultative anaerobes or aerobes Which has worst prognosis? Ledge in midroot, fracture while obturating, transport through apex in curved root

Temperatures for autoclaves is governed by FDA. For carbide stainless steel burs, what form of sterilization does not corrode the surface? o Dry Heat, 2. Steam , 3. Both Dry Heat and Steam, 4. Neither Dry Heat nor Steam To improve the quality of radiographic film, what parameter would be altered? o 1.mA, 2. kVp, 3. Exposure time, 4. o Less Kvp low contrast for perio, mA is quantity

What bacteria is involved in the carious process but NOT an intitator? o Streptococcus, 2. Lactobacilli, 3. Actinomyces, 4. Staphylococcus o There is a question about what is involved in the initiation of caries. Lactobacilli is in 3 choices so if you know it s not an initiator, you can deduce the answer Which tooth is least successful for a patient to remove plaque with floss? o 1. Maxillary premolar, 2. Mandibular premolar, 3. Mandibular molar, 4. Maxillary molar A study failed to report 5 cases of caries. What is this called? o 1. True Positive, 2. True Negative, 3. False Positive, 4. False Negative Upon extraction of a maxillary molar, a 2mm sinus defect is noticed. What is the next step in treatment? o No treatment necessary, follow up in 3 days. o Which premolar is most likely to have 3 canals

A teenager (11 yo) presents with an intruded maxillary lateral incisor (#7). What is the next step of treatment? o Extract and Implant, 2. Splint for 2 weeks with RCT treatment after, 3. Splint for 2 weeks and follow up o 4 year old intrudes primary maxillary central, what do you do leave it, wait for it to re-erupt For a young patient (6 yo), the treatment of choice for a necrotic pulp on permanent first molar would be: o 1. Apexification (open apex, necrotic) 2. Apexogenesis (open apex, vital) 3. Root Canal Treatment (closed apex) o It takes 10 yrs from calcification for root completion, 13 years for canine Which treatment has the least successful long term prognosis on a deep carious lesion on #3? o Direct pulp capping, 2. Indirect pulp capping, 3. Pulpotomy, 4. Pulpectomy and RCT What would be the most reasonable cause for a tooths symptoms to change from reversible to irreversible pulpitis? o Accumulation of traumatic injuries, 2. Bacterial involvement inside pulp chamber, 3. Increased intra-pulpal pressure What is the most accurate pulpal test to determine vitality of a tooth with a full-gold crown? o Electric testing, 2. Percussion test, 3. Palpation test, 4. Thermal test What is the outline shape of the access cavity for a permanent mandibular first molar? o Triangular, 2. Trapezoid, 3. Oval, 4. Circular On a radiograph, you notice a distal carious lesion on the mandibular first premolar. Upon restoring the adjacent 2 premolar, with direct vision, you notice that there is no decay. o What is the most probable cause? 1. Cervical burnout, 2. Root caries For a lesion in enamel that has remineralized, what most likely is true? o The enamel has smaller hydroxyapatite crystals than the surrounding enamel o The remineralized enamel is softer than the surrounding enamel o The remineralized enamel is darker than the surrounding enamel o The remineralized enamel is rough and cavitated o Know about chalky white lesions On the cementation day for a full gold crown, what is the first step? o Evaluate margins, 2. Evaluate occlusal contacts, 3. Evaluate proximal contacts, 4. Evaluate proximal contours For a surgical extraction of mandibular first molar, what is the best course of action? o Cut a Y into the coronal portion and extract each root separately, o Cut the coronal portion from the roots and extract the roots separately, o Cut the tooth bucco-lingually and extract the roots separately
nd

For a surgical extraction, what does not contribute to developing post-operative pain? o High-speed drill, 2. Low speed low torque drill, 3. Sharp burs, 4. High-speed drill with water spray Onlay wall preparation: o Wall:271 carbide bur, 2-5 degree taper, axial walls divergent o Box: cavosurface margin 30-40 degrees o Bevel: diamond, 0.5mm width, want 40 degree gold margin on occlusal and groove extension, 30 degree on cusps o Skirt: extends casting aaround line angle to increase retention and resistance form pulse n bp what kind of measurement - nominal, ordinal, interval, ratio temperature Celsius = interval. Kelvin = ratio. apirin - single dose - how much time- 4 hours, 1 day, for baby aspirin dosage is 81mg/day Adenomatoid odontogenic tumor o Young females, maxilla o Radiolucency around an unerupted tooth extending past the CEJ. o It should be differentially diagnosed from a dentigerous cyst and the main difference is that the radiolucency in case of AOT extends apically beyond the cementoenamel junction. Sore muscles in the morning: o Myofacial pain dysfunction syndorme

KNOW THIS! Injury Fractures Enamel fracture class I Enamel/Dentin class II Crown w/ pulp class III

Closed apex Smoothen, restore lost tooth structure. Good px Restore with bonded resin. Good px. Vital pulp therapy if possible If not, RCT (also if complex restoration needed) Dx with occlusal + 3 PA (at different angles) Vital: vital pulp therapy + splint Coronal: splint 6-12 wks, if no reattachment, EXT Midroot: splint 3 wks, endo if possible, EXT Apical good px, pulp should be vital Occlusal adjustment, no tx, follow up Splint 3 wks if mobile Resposition, splint, endo if necessary Extrusion: 65% pulp necrosis Lateral: 80% pulp necrosis Closed apex: ortho/surgery, endo 96% pulp necrosis Re-implant immediately (within 0-60min) Clean socket with saline (no curet), replant Splint 7-10 days Pen or Doxy for 7 days refer for tetanus booster Re-implant (after 60 min) 2.4% NaF for 5 minutes Saline, Replant, splint 7-10 days, Pen or doxy 7d Endo 7-10 days after replantation in both cases

Primary teeth or Open apex Smoothen enamel, check vitality 1,2,6 months Smooth/restore Primary: Vital: pulpotomy, Necrotic: pulpectomy Resorption: EXT Relatively rare Coronal : splint or EXT Apical : no tx

Horizontal root fracture

Luxation Class V Concussion percussion + Subluxation mobility + Extrusion/ lateral luxation Intrusion

Avulsion class VI

No tx, CHX if desired, Better px than closed apex No tx, CHX if desired, Better px than closed apex If before periapical blood clot reposition and splint 7-10 days Then endo No tx unless determined that tooth is impinging on permanent successor; should re-erupt Within 0-30 minutes Clean root with saline if contaminated Tooth in doxy, clean socket Replant, splint 7-10 days, soft diet Pen or doxy (if not succeptible to tetracycline staining); tetanus booster After 60 minutes replantation not indicated

7-10 days later try apexification

Ginseng - antiplatelet (interferes with coagulation not given with aspirin). In natural teeth 2 types of hue yellow and yellow red o Yellow stain increases the chroma in the yellow hue o Orange stain increase the chroma in the yellow red hue. o Pink purple changes the hue of the teeth. Move from yellow to yellow red o crown unesthectic value Width of periodontal ligament 0 .2mm perodontal stability attachment loss is the most imp criteria in diagnosing periodontal conditions, followed by mobitiliy. Flaps o Partial thickness (mucosal) flap only epi and CT o Full thickness includes periosteum Horizontal incisions all three together allow for visualization of alveolar bone Internal bevel incision: o 0.5-1mm from fgm apically displaced flap o 1-2mm from fgm modified widman flap (not reflect beyond mgj) o Coronal to base of pocket undisplaced flap or reverse bevel incision crevicular incision: o Base of pocket to crest of alveolar bone, conserves papilla; o (1) + (2) creates a collar of tissue around teeth interdental incision: separates collar of gingiva from tooth Cover site with periodontal pack dont enhance healing rate though. Contain zinc oxide and some with Abts Give CHX 2x/day for 1-2 weeks

Bupropion (Wellbutrin, Zyban) is an atypical antidepressant and smoking cessation aid. o It acts as a norepinephrine and dopamine reuptake inhibitor, veracity - being truthful to patient dentist doesnot complete the treatment - pt abandonment protection act- Good Samaritan Act is accepted in all states, but dentists are not included in all the states. Collimation does everything except o reduce pt exposure, reduce operator exposure, film fog, reduce average energy of x-rays

The Dentist completes exam and advises x-rays but the pt refuses. What should the dentist immediately do. o Explain the need for X-rays. What is true of osteoradinecrosis. ANS. Commonly affects mandible X-ray identification: Median Palatal Suture, Zygomatic process of maxilla, Dorsal Surface of tongue, odontoma Radiology The severity of response increases with the amount of X-ray exposure. This effect is called: Deterministic, Stochastic, Genetic?? o Radiosensitive cells ANS. Hemopoeitic bone marrow. o When there is no barrier, protection of dentist. 6 feet; 90-135 degrees o If the buccal root of maxillary ist premolar appears distal to palatal root .The X ray was mesially angulated o Reverse occlual plane on a panoramic view. Chin tilted upwards; smiley(happy face) is chin down. o Best view to observe pathology of maxillary sinus CT , Panoramic, Lateral skull view ( all these are good to view max sinus) waters

What is the reason for increased radio-opacity on the mesio-occlusal surface of maxillary I molar. Cusp of Carabelli, restoration on disto-lingual surrface, dens in dente

Transillumination is useful in the diagnosis of .Class 1, class 2, class5, class3 Commonest tooth with vertical root fracture .Maxillary incisor, mandibular incisors, maxillary molars, mand.molars. Which tooth refers pain to the ear ANS. Mandibular molar Referred pains: Max Incisiors Naso labial, Canine Frontal region, Premolars Temporal region, Mand. Molars Ear and infratemporal region. Which are the two most imp. steps for diagnosis. History and clinical examination Which procedure does not require antibiotic prophylaxis o Non surgical endodontic treatment, Scaling and root planing, Extraction, surgical treatment. Material used for mouth guard vital bleaching ANS. 10% carbamide peroxide. Most critical for pulpal protection ANS. Remaining dentin thickness 2mm Critical factor determining the success of implantation of avulsed tooth - time Tooth with the best prognosis ANS. Internal resorption Q about characteristics of internal resorption What is the effect of extraction of primary 2 molar on the eruption of 2nd premolar with one third root formation . o Eruption is faster. Eruption is slow, no effect on eruption rate, condition varies depending on the patients age. Which structure exhibits synchondrosis o Where the connecting medium is cartilage, a joint is termed a synchondrosis. o An example of a synchondrosis joint is the sternocostal joint (where the first ribs meet the sternum). In this example, the rib will join up with the sternum via the costal cartilage.)

What is the effect on growth of a child with unilateral sub-condylar fracture retarded growth Gingival index is an example of .ordinal scale, nominal scale Hybrid layer. ANS primer within intertubular dentin The provoking factor for pain after placing bridge. Heat, cold, sweet, biting(occlusion) Kerrs syndrome Kerrs syndrome shows all except Maxillary ridge resorption, flabby tissues, enlarged tuberosities, increased vertical dimension of occlusion. Lack of function of indirect retainer is manifested by ANS. Lifting of distal extension away from tissues. Common feature between porcelain veneer and all-ceramic crown preparation rounded internal line angle How do you decrease the width of artificial teeth. Deepen the facial line angle proximally and increase the interproximal embrasure Deepen the facial line angle proximally and decrease interproximal embrasure take the facial line angle labially and increase the interproximal embrasure take the facial line angle labially and decrease the interproximal embrasure The dentist cements the porcelain veneer with light cured resin and the patient returns with brownish discoloration at the margins.why? o not enough cement or microleakage(depends on duration of pt return)

patient receives a blow to the chin who has a MOD inlay placed on the maxillary molar 3 months earlier.Now the patient has a vague pain on biting ,there are no other symptoms.why? maxillary sinusitis, m-d fracture, b-l fracture when you wax the removable partial denture on a cast; what is it called- Master cast, refractory cast Purpose of addition of tin and indium to metal ceramic alloys ANS.chemical bond-covalent bond Cause for post-operative sensitivity of composites - shrinkage Something about how soon do you see the marginal leakage or discoloration ? The condylar guidance is increased from 20 to 45 degrees,what do you do. o Increase the compensatory curve o the curvature of alignment of the occlusal surfaces of the teeth that is developed to compensate for the paths of the condyles as the mandible moves from centric to eccentric positions. o A means of maintaining posterior tooth contacts on the molar teeth and providing balancing contacts on dentures when the mandible is protruded. o Corresponds to the curve of Spee of natural teeth. curve of Spee o an anatomic curvature of the occlusal alignment of teeth, beginning at the tip of the mandibular canine, following the buccal cusps of the natural premolars and molars, and continuing to the anterior border of the ramus, as described by von Spee. the curve of the occlusal surfaces of the arches in vertical dimension, brought about by a dipping downward of the mandibular premolars, with a corresponding adjustment of the maxillary premolars. curve of Wilson o the curvature of the cusps, as seen from the front view. The curve in the mandibular arch is concave, whereas the one in the maxillary arch is convex. Reverse curve o a curve of occlusion that is convex upward when viewed in the frontal plane. What is rest position muscle guided Centric occlusion is tooth guided, centric relation is ligament guided. You are giving cusid-cuspid bridge and want to change the canine guided anterior disclusion.What is required. o change to group function All are requirements of interocclusal record material except. o Hard when set, capable of trimming with sharp knife after set, resistance to biting. All are symptoms of TFO on an implant except. Gingivitis, pain, loosening of implant, breakage of abutment screw. Heart rate of Age 3 is 110, age 5 is 100, age 12 is 75 and adult is 70 o Resp rate: 25, 20+, 20-, 15 Greatest incidence of malignancy is seen in Pagets disease, monostotic fibrous dysplasia o malignant change ,usually development of osteosacoma has been RARELY associated with fibrous dispalsia. o radiation for this lesion is contraidicated because it carries the risk for developmant of post-radoiation bone sarcoma o Development of malignant bone tumor, usually an osteosarcoma is recognized complication of Paget, frequency of osteosarcoma is 1%,mostly long bone. Oral granulomas, apthous ulcer, rectal bleeding is seen in. Wegeners granulomatosis, ulcerative colitis, crohns disease. Pierre-robin syndrome consists of a triad of retrognathia, glossoptosis, cleft palate. Most important factor in shade selection. ANS-value

Flexibility of a clasp arm depends on all except. Length, taper, circumference, depth of undercut, Which is most rigid .pd-Ag, typeIII gold,typeIV gold. Least wettability is seen with which impression material.polysulfide, condensation silicones, hydrocolloids, polyether. Most stable in moisture environment. Polysulfide, condensation silicones, addition silicones, polyether. Setting of polyvinylsiloxanes is retarded by. ZOE, latex gloves, ferric chlorides, aluminium chloride Which produces least change on implant surface while removing calculus. Ultrasonics, sonics, curettes, plastic curettes What has ultimate effect on the thickness of epithelium of free gingival graft. Recipient epithelial tissue, donor epithelial tissue, donor CT, recipient CT Disadvantage of partial thickness flap are Visibility, access??? The purpose of GTR is to prevent. Long JE, migration of PDL cells, Migration of CT cells. The resorption of bone in PD disease is caused by.IL1, IL8, IL10 o IL 8 chemotaxis, IL 10 is macrophages. The biological width is 2mm,3mm, 4mm, 1mm, 5mm. The internal bevel gingivectomy extends approximately from Indications for gingivectomy gingival hyperplasia, The purpose of barrier. Apical movement of PDl cells, Coronal movement of cells. Gingivectomy is contraindicated in .The sulcus is apical to gingival groove, sulcus is apical to convexity of tooth, sulcus is apical to the crest of alveolar bone. periodontal pathogens in health- ANS Gram+ facultative cocci and filaments. Desquamative gingivitis is associated with which 2 conditions. Pemphigus and bullous pemphigoid, Lichen planus and erythema multiforme o Pemphigus supra, acantholysis o Pemphigoid - basal o Questions about lichen planus The depth of sulcus is 5mm the distance between CEJ and the base of sulcus is 2mm.what is the attachment loss. 2mm The role of chlorhexidene is cos of.ANS- Substantivity high concentrations for long time Condition seen in normal individuals and also in pregnant patients.ANS Pyogenic granuloma or pregnancy tumor. Antibiotic seen in GCF- ANS Doxycycline, minocycline First thing u need to check while placing crown.ANS Esthetics then Proximal contact Miller classification for recession o Class I: recession doesnt extend to mgj, no bone or soft tissue loss interproximally good to excellent px o Class II: recession on or beyond mgj but no bone or soft tissue loss interproximally good to excellent px o Class III: recession on or beyond mgj, with bone and soft tissue loss interdentally partial coverage result o Class IV: recession on or beyong mgj, severe bone and soft tissue loss interdentally poor prognosis

Mechanisms of healing after perio surgery o Regeneration: growth and differentiation of same type of tissue that was damaged by disease o Repair: healing by SCAR o New attachment: embed new pdl fivers into new cementum and attachment of ging epi to a previously diseased root surface

Implant vs. Tooth o No PDL, no supracreastal CT insertion into the implant o May probe to the level of the bone (no JE to stop you) o Will not erupt so dont use in growning individuals o Complications Screw loosening (posterior > anterior), implant fracture (<1%) Peri-implant mucositis = gingivitis Peri-implantitis = periodontitis Periostat = subantimicrobial-dose doxycycline (SDD) selectively inhibits MMP-8 and MMP-13 destruction of collagen. o Approved as adjunct to SRP tx of chronic periodontitis o 20mg dose 2x/day for 3-9 months; CI: preggers, kids under 12, tetracycline allergy Perio procedures o Gingivectomy elim SUPRAbony pockets, gingival enlargements, suprabony perio abscess CI: osseous recontouring, pocket is apical to mgj, inadequate attached ging, esthetic concern Bevel incision apical to pocket depth Healing by SECONDARY INTENTION (clot, epithelial migration, ct repair o Gingivoplasty reshape tissue: clefts, ging enlargement, shelf-like papilla CI: reduce or eliminate periodontal pockets o Mucogingival surgery widen attached gingiva, deepen vestibule, resection aberrant frena via graft and AP flap o Osseous Surgery Correct osseous M and D crater by recontouring F and L walls Correct vertical or angular defects by resective osseous surgery or perio regeneration Ressective osseous surgery for interprox bony craters, early furcations, cases w/ thick alv bone CI: esthetics Ostectomy removing tooth-supporting bone; must be sure to remove widows peaks Osteoplasty removal of nonsupporting bone o Regeneration GTR prevent epi migration on cementum while pocket is healing from flap reflection Membrane used to exclude epi and CT from root and protect clot formed Citric acid and/or Emdogain enhance new attachment of ging tissues following excision o Implants Titanium implants offer the best biological attachment to bone and gingival tissue osteointegration The normal recall appointment between periodontal treatment. 3 months, 1 month, 4 month, 6 month 45 year patient comes for appointment schedule.His B.P is 160/100.What should the dentist do. o Call up the physician, reschedule appointment when the B.P comes down , check B.P again after 15 mts Frequent urination seen during 3 trimester ANS. Pressure on the bladder. Curettes used for distal surface of maxillary II molar ANS. #13-14 Pt comes with carious involvement and localized swelling of cheek.What is the immediate treatment. establish drainage After periodontal surgery, the dentist leaves interproximal bone apical to radicular bone.What is this called o negative architecture. Evaluation after scaling and root planning is done after how many days. 7-10 days, 14-21 days 4-6 weeks
rd

Order of tx: emergency, disease control, re-eval, definitive, maintenance Mode of action of ultrasonics. ANS. Vibration in elliptical (magnetostrictive) , sonics is linear(work with air) Which of the following is not an advantage of Ni-Ti over stainless steel file. Maintains the shape of canal, flexibility, resistance to fracture. Best theory to explain dentin hypersensitivity. ANS Hydro dynamic theory Gastric acids cause. ANS. Erosion Primary effect of an over hang. ANS interferes with plaque removal Medications associated with hyperplasia. Calcium channel blockers, Dilantin sodium, Cyclosporine o Nifedipine In most of the cases, localized fibromas are often: Dysplasias, metaplasia, anaplasia, hyperplasia. The major environmental risk factor for periodontal disease. Diabetes, smoking Which is not true of elder abuse. o Most of the elder abuse is at victims home o Mostly it is by victims relative, o Elders abuse is often over reported and exaggerated o un-authorized use of ATM card is some times considered crime but not abuse Most common condition affecting elders. ANS Depression All are seen in DI except. o Large bulbous crowns with short tapered roots o large irregular dentinal tubules o accentuated DEJ o opalescent hue In a scientific article, where should the description of study sample be? materials and methods, results, discussion Common form of leukemia in children Acute lymphocytic leukemia o acute leukemias respond to tx well, chronic doesnt . o AML is worst prognosis Viral load for HIV pts test sensitivity detect more than 48 viral copies., o Neutrophils less than 500 no tx o Platelets below 50000 no tx o CD4 should be 200, but 50 is critical. Increased no.of teeth + supernumerary and impacted teeth is seen in ANS. Cleidocranial dysplasia. o Gradners syndrome has supernumerary teeth. When should the dentist not use para-phrasing. o When trying to speak to a patient in his second language o When the dentist is upset with what patient says o when giving factual values. o How does patient determine if pt can sign concent forms? The patient who is diagnosed with Basal cell carcinoma, says to the dentist give me the report, do I have cancer, what should the dentists initial reaction be. Pic of basal cell carcinoma, white old man, outside of mouth o DO you have any one accompanying you, this cancer has better prognosis than other cancers.

Which of the following is not recommended for a patient who is on Nicotine de-addiction o mucous patches, nicotine gum, Bupropione, nicotine nasal sprays. o Buproprione is also used to treat depression Which of the following does not have cauliflower like , pebbly appearance. o Verrucous carcinoma, fibroma, condyloma accuminata, papilloma. o Description of papilloma Common finding in a patient with cerebral athetoid palsy. ANS. Anterior teeth fracture Anti-fungal used as troches for treatment of oral candidiasis.ANs Clotrimazole o swish and swallow is nystatin. o AmphotericinB is only antifungal given in IV o Fluconazole(tab) is given for systemic candidiasis. The direction of healing of wound after extraction of tooth in mandibular arch o outward and upward, inward and down ward Infections from mandibular premolars spread to which space Submandibular, because the roots are below the mylohyoid. Anterior roots above the mylohyoid has submental spread. Cellulitis most of the time involves unilateral, ludwigs angina is bilateral and complication is edema of GLOTTIS. The patient who is a tobacco addict says to the dentist,I want to quit the habit,what should the initial reaction of the dentist be. All are seen in a patient with sjogrens syndrome except: sicca, xerostomia, lymphoma, arthritis, other choice is option. Amount of fluoride supplementation required in 16 yr .0 mg The greatest incidence of dysplasia and carcinoma in situ erythroplakia Clinical photographs- Lymphoepithelial cyst, cheek biting, smokers palate, Dentinogenesis imperfecta, facial palsy, migratory glossits, mucocele, fordyces granules Dentist realizes that the distal margin of crown preparation is within 1mm of interdental bone.what should the dentist do. ANS crown lengthening Necrotizing sialometaplasia minor salivary gland disease presents on the palate which is most commonly confused with carcinomas due to the ulcerated presentation. Heals without scarring. Transillumination in children is used for sialolithiasis Questions on herpes ulcers and apthous ulcers The patient develops oral ulcers and target or bulls eye skin lesions within 24 hrs.ANs Erythema multiforme Suppuration is seen with ANS.Actinomycosis sulphur granules, seen in the neck. A chronic alcoholic requires extraction what test should the dentist advise for INR which of the following is not true for a patient requiring surgery and on oral anti coagulants. o PT should be 1.5-2 times that of control, INR should be above 2.5 o Norma INR is 1, can be treated till 3(acc to manual) Question on parulis o bumps on the gum parulis o Pyogenic granuloma, peripheral gaint cell granuloma, peripheral ossigying fibroma, peripheral odontogenic fibroma,

A patient received radiation therapy and requires extraction,what should the treatment be.Extraction, extraction with alveoloplasty and sutures, extraction with alveoloplasty of basal bone and suture, pre-extraction and post extraction hyperbaric oxygen Virus associated with Chicken pox also causes ANS herpes zoster Unilateral lesions after herpes zoster infection. Reactivation of the virus from sacral ganglion causes shingles o induced by stress and sunlight exposure. Fluoride supplement required in a 2.5 yrs child in a non-fluoridated area 0.25mg know supplement chart Community water fluoridation 1ppm, school water fluoridation 4.5ppm A 4 yr old child management empathy and respect Management of moderately apprehensive child Replacing words like LA with sleepy juice is called as Euphenism. Use of praise, smile and appreciation is-Token(positive) reinforcement, social reinforcement What is not an advantage of rubber dam when compared to not using it. Improved properties of materials, shortens operative time, facilitates the use of water spray Closest resemblance of deciduous mand II molar permanent mand I molar The success of implants does not depend on Age of the patient Prilocaine above 500mg causes: Apnea, cardiogenic shock, methemoglobinemia Complication associated with removal of internal oblique ridge (mylohyoid) ridge. ANS Lingual n The drug which causes withdrawal symptoms in pts taking oxycodene The drug contraindicated in pt taking ginkgo biloba. Heparin (CI with all antiplatelet/anticoagulants) o Ginseng used for male impotency, diabetes o Ginkgo biloba used for memory loss. The drug of choice in patient with bradycardia - Atrophine, Epinephrine. Which of the following does not have anti-inflammatory action. ANS Acetaminophen Some question on difference between aceta and aspirin but it was about about anti-inflam action. Something about duration I think The immediate choice of tx for large radiolucency in the mandible. Biopsy, aspiration biopsy and wait for the bone to fill in o Safest of all biopsies aspirational The common symptom of trismus is associated with which space involvement. o Sub-masseteric, superficial temporal, During IAN block, pierce Buccinator muscle and inject in pterygomandibular space. The patient returns to the dentist the next day after extraction with pain and swelling. o The drug of choice in this penicillin allergic pt. Clindamycin 300mg qid The restraining of uncooperative 2 yr child should be done by Dentist, Assistant, Parent Why shouldnt patient be in the room? The information about hazards of chemicals used in the office should be present in ANS. Material safety data sheets o Red flammable o White Personal protection o Blue health hazard

Yellow- identitifies the reactivity or stability of a chemical.

The failure of a test to detect the presence of 5 cases of disease. ANS. False negative The dentist separately for core-build up and the crown but the insurance company says that the core build up is part of crown.what is this called. bundling know unbundling also the investigator studies the occurrence of oral cancer in pts in a private nursing home. what is this? o Cross-sectional study, longitudinal study, Descriptive epidemiology(prevalence and incidence) common missing permanent tooth ANS. Maxillary third molar also mand second pm, maxillary lateral incisor The main role of chlorhexidene before surgery ANS. Reduce the no.of microbes

Blood transfusion before surgery should be done when the platelet concentration falls o below. 20,000, 50,000 , 100,000 Crowing sounds are seen with. Acute asthmatic attck, COPD. o Asthma Exhalation wheezing o COPD expiration wheezing. o Croup------Barking Cough The most effective method of caries reduction. ANS. Systemic water fluoridation 5 As of cancer prevention : Ask, Advise, Assess, Assist, and Arrange The Child has 12 deciduous and 12 permanent teeth.what is the age of the patient 8-9 yrs When does the calcification of permanent incisors occur 3-4 months(decks) Excepting maxillary lateral incisors 10-12 months The drug of choice in status epilepticus Diazepam The bacteria that causes rheumatic fever.streptococcal infection like sore throat or scarlet fever What is the correct method of excavation of deep caries. Long bur from periphery to the center Purpose of post .retention of core Threaded post, what is the best way to prevent root fracture when inserting? Apical closure of permanent tooth occurs. ANS.21/2 to 31/2 yrs after eruption The dentist realizes that there is formation of ledge.what should he do next.ANS Gently by pass the ledge. Pt asks questions regarding the face mask, gloves and other sterilization methods.what should the dentist say. probably something to do with standard(universal) precautions The patient retires and loses health benefits. treatment is done on the next day. the pt requests the dentist to enter the previous day date and the dentist does Fraud The frequent intake of juices by the child does all except. ANS. Increase ph., juices decrease PH Child goes to bed with bottle filled with ________________, which one doesn t cause caries? o Choices were formula with F- water, plain non F- water, 100% juice, and something else Which is true of intra pulpal anesthesia? produces anesthesia after 30 sec, it does not cause the discomfort to patient, produces anesthesia by pressure.

The position of permanent incisors in relation to deciduous incisors. ANS.inferior and lingual Advantage of distraction osteogenesis over ostectomy . better patient compliance, wider range movements, less relapse and can be done in yonger patients What is not true of alveolar osteitis (dry socket). ANS pain starts on the first day, curettage should not be done in pts with dry socket, no antibiotics. Acid-etching does not cause. Reduced leakage, better esthetics, increased strength of composites. Purpose of placing posterior palatal seal ANS.compensates for shrinkage. o Factors to consider when designing posterior palatal seal Patient comes back to the clinic 5 hrs after extraction with bleeding. what should the dentist do. ANS.search for the source of bleeding flexibility of the clasp depends on all except. Depth of the undercut, cross-section of the clasp arm, taper of the clasp, length of the clasp. Pt after scaling and root planing and with excellent oral hygiene does not show reduction of pocket depth, what is the next step for the dentist to do.ANS. Periodontal surgery. Common location for periodontal cyst. ANS. Lower bicuspid o lateral periodontal cyst is detected only during radiographic examination o 65% of cases occur in mandibular canine-premolar area. Patient has 2mm communication with the maxillary sinus. what is the treatment of choice. o 2mm: dont do anything and follow up o 2-6 mm: place gel foam (surgicel), suture ,decongestant and antibiotic , inform patient o more than 6 mm: buccal flap How to split the impacted mandibular tooth when viewed from occlusal.+, Y slope, split buccal to lingual till the furcation. Potency response to a drug over a given range of concentrations. o Potent = depend on dose of drug o less mg for same efficacy has more potency efficacy the max effect of the drug. Max effect is also called as intrinsic activity. Therapeutic index - is an estimate of the margin of safety of a drug. o TI = Lethal dose 50/Effective dose 50 What is bioavailability of a drug. - amount of drug that is available is blood. What is the effect seen when propranolol and epinephrine are injected simultaneously o in cases of mild reactions it causes hypotension; in severe reaction it is malignant hypertension Synergism - same target of action-increase action o additive - target of action is different Patient c/o burning sensation at the corner of the mouth - angular chelitis 6yr old complaining of halitosis o can be a symptom of various conditions including: postnasal drip, dry mouth, dental problems, abcess, and sinusitis. o often due to the decomposition of mucus secretions and debris which accumulate on the tongue, in the nose, and between the teeth.

Patient c/o frequent dislodgement of anterior crown (post and core) Extension of lower denture - till retro molar area : o primary support area on mandible: Buccal shelf , o primar support area on maxilla: Ridge, secondary on max - rugae. What incision is given to remove palatl tori - double Y incision was not the choice ; o other possible answer might be a linear incision from A-P with oblique releasing incisions How do u clean implants - scaler with a plastic sleeve; rubber cup and paste, stailess steel scaler, titanium scaler It is required/mandatory to report all except - child abuse, reaction to drug, one more choice, o Abuses that have to be reported to authorities colleague practicing with chemical impairment colleague advertising on electronic media child abuse domestic violence elderly abuse Listerine - Phenolic compound - Listerine is a combination of alcohol + phenolic compound. o Burning after use of mouth wash is due to alcohol (causes drying of the mucosa). Patient had old PFM on # 9, he complains that PFM is lighter than other teeth, what will u do? - change PFM, bleach teeth o When will you bleach teeth in anterior veneer prep- before veneer prep, wait for 2-3 weeks o bleaching affects both enamel and dentin incisal guidance - guided by overjet and overbite IRM is - what is the compoisition - zinc oxide-eugenol with polymer reinforcement. where will you place the margins in a anterior PFM prep - Subgingival, at the crest PVS impression material has which of the following - is unstable, doesnt come out completely from tissue undercut, releases alcohol as byproduct, may release H ion Mostly periodontits occurs in : Diabetes, Tobacco St John Wort(herb) is used for - mild depression Drug that interferes with coagulation - Saw palmetto o Saw palmetto products could theoretically interfere with blood coagulation, so concurrent use with blood thinning medications, such as coumadin (warfarin,aspirin and other NSAIDs) should be avoided. Hyperthyroidism restless/irritable, heat intolerance, weight loss, muscle wasting, tremor, diarrhea, sweaty, fine hair o Causes: Graves, Thyroid adenoma, Pituitary adenoma (2 hyper), Hashimotos thyroiditis (early) Hypothyroidism mental slowing, cold intolerance, weight gain, deep voice, constipation, edema, dry skin, hair loss o Causes: Cretinism (child), Myxedema (adult), Hashimotos thyroiditis (late; autoimmune) Hypoglycemia is common in : Hypotension, Hyponatremia, Hypokalemia Drugs used to control salivary secretion Pilocarpine, cevemille - increases, Atropine - decreases

To expose a mandibular lingual torus of a patient who has a full complement of teeth, the incision should to o a. Semilunar b. Paragingival o c. in the gingival sulcus and embrasure area o d. directly over the most prominent part of the torus o e. inferior to the lesion, reflecting the tissue superior

ARCON ARTICULATOR - An instrument following anatomic guidelines such that the ball of the condyle analogs are carried on the mandibular element, and the fossa assemblies on the maxillary element. CONDYLAR ARTICULATOR - A non-arcon articulator. Advantage of Arcon over Condylar Articulator o When using an occlusal record to relate opposing casts, the thickness of the record increases the VDO upon articulation. o When the record is removed the condylar guidance inclination would be decreased in the non-arcon articulator creating a potential negative error in excursive movements. Pt has a stable bite, ant. guidance is fine and you need to fabricate FPD. What kind of articulators? - Semi adjustable. Non-Adjustable o (Class 1) A simple holding instrument capable of accepting a single static registration. Vertical motion is possible. o (Class 2) An instrument that permits horizontal as well as vertical motion but does not orient the motion to the tmj o Uses: Single restorations, multiple restorations if good bilateral cuspid disclusion exists; Certain FPDs. Semi-Adjustable o (Class 3) An instrument that simulates condylar pathways by using average or mechanical equivalents for all or part of the motion. o These instruments allow joint orientation of the casts and may be arcon or nonarcon instruments. Non arcon: Hanau H2, Arcon: Whip Mix o Uses: Multiple restorations; Fixed partial dentures with: minimal occlusal pathology, no loss of VDO, no immediate side shift, Full mouth restoration if good anterior guidance exists. Fully-Adjustable o (Class 4) An instrument that will accept three dimensional dynamic registrations. Those instruments allow for orientation of the casts to the temporomandibular joints. o Uses: Full mouth restoration, Extensive occlusal pathology, Group function/posterior guidance, Restoring at a different VDO Purpose of making plaster index of complete denture - to preserve face bow record Why do we remount complete denture? o The first remounting is performed before the dentures are handed in to the patient and the second after a certain time of wearing. o The first procedure corrects occlusal errors resulting from imperfect fabrication o The second, after wearing of dentures, adjusts the denture base on the denture foundation. Lesion that blanches - Leukoedema - it disappears on streching, it is always bilateral. Apexification (necrotic pulp) and apexogenesis (vital) Tooth responds with lingering pain to cold - irreversible pulpitis with normal periradicular tissue; irreversible pulpitis with chronic periradicular tissues Hallmark of periradicular abscess - sinus tract Hallmark of acute periradicular periodontitis - sensitivity to percussion Rubella embryopathy - also called chronic rubella panencephalitis o Occurs during 1st trimester. Fetus has microcephaly, deaf, cataracts, congenital heart disease, purpura Periodontium surrounding the implant - no periodontium, bone and implant. Differentiate perio and pulpal abscess- Endo to perio lesion is outward, Perio to Endo lesion is inward. Which has best px? Perio lesion caused be endo, endo lesion caused by perio, combo lesion, perio caused by perio

Reliable test for teeth with open apex - heat, cold, ept--- same when tooth has gold , or porclain crown Organisms in chronic periodontitis P. gingivalis: o Aggresive perio - Loclaized: AA, Gen is Prevotella intermedia, Eikenella corrodens Midline swelling all except - globulomaxillary cyst (doesnt really exist but is a RL between maxillary lateral and canine) o midline swellings - nasopalatine, median rhomboid, thyroglossal duct, dermoid cyst granuloma + bleeding + apthous ulcer -: ulcerative colitis, chrons disease how to prevent proximal displacement of Cl II filling - retention grooves how to increase the resistance and prevent from fx of the restoration: dove tail it provides the retention form Coefficient of thermal expansion is most for which material tooth < gold < amalgam < filled resin < unfilled resin Epi in each ml of lido with 1:100,000 0.018mg per carpule o Max dose of epi for cardio pt----- 0.04mg, Two carpules o Max dose of epi for healthy pt---- 0.2 mg, Eight carpules

Rule of 6s o F- > 0.6 ppm NO SUPP o Pt < 6 mos NO SUPP o Pt > 16 yrs NO SUPP 2.2 mg of NaF will provide 1 mg of Flouride *memorize* Tooth release of pain on biting and sensitive to cold - cracked tooth syndrome o Tooth with a recent crown - sensitive to cold and biting ( hyperocclusion) Diagnosis of ANUG - punched interdental crater on the interdental papilla, fetid odor, metallic taste and pseudomembrane. Reason of cheeck biting in a denture patient - when mandibular molars placed more buccally IF SNA is -8 what type of malocclusion---Class III o SNA position of maxilla; normal = 82 o SNB position of mandible; normal = 80 o Convex profile Class II o Concave profile Class III o ANB >4* then Class II What is ANB? Relation of maxilla to mandible o Cl 1 average 2 ; range 0-5 degree o Cl 2 average 7 ; range 2-12 degree o Cl 3 average -1 ; range -6 -0 degree What will happen if pt consumes too much of opiod analgesic - respiratory depression pic of cyclosporine (probably gingival hyperplasia) complication following distraction osteogenesis : - long term followup, nerve damage NaOCl does all except - chelating agent

Pharmacology o Overdose of PRILOCAINE - Methemoglobenemia o Lidocaine toxicity is due to - overdose - increased vasoconstrictor, due to preservative o Drug with no inflammatory action - acetaminophen o Antidote for benzodiazipine overdose Flumazenil (Anexate) o Antidote for opiods Nalaxone o Treat alcohol addiction Disulfiram If dentist seals a small carious lesion what happens - caries stops, caries increases Normal Dose o Amoxillin - 500mg TID o Clindamycin - 300mg qid, longterm usage is pseudomembranous colitis o Cefalexin - 500mg qid Premedication single dose, 30-60 minutes before procedure o Oral: Amox 2g; 50mg/kg o IM or IV: Ampicillin 2g; 50mg/kg OR Cefazolin/Ceftriaxone 1mg; 50mg/kg o Pen allergy: Clindamycin 600mg; 20mg/kg OR Cephalexin 2g; 50mg/kg OR Azithromycin 500mg; 15mg/kg o IM or IV: Clindamycin 600mg; 20mg/kg OR Cefazolin/Ceftriaxone 1mg; 50mg/kg When will the BULL rule be utilized with the selective grinding working side ONLY Pan showing lucency going inferior over the body of mandible close to the angle. Informed the patient was involved in an accident. Identify the lucency fracture There are more detached plaques within supragingival plaques that subgingival plaques. The detached plaques within subgingival area are the ones that are more toxic to tissue than attached plaques o both statements are correct o the first statement is correct but not the second o the first statement is wrong, but the second statement is correct - supragingival plaque is always attached and subgingival is unattached. o both statement are wrong Organism that cause severe spreading abscesses include: Fusobacterium, Campylobacter, Enterococci, Bacteroides Definitions o Unbundling: "the separating of a dental procedure into component parts with each part having a charge so that the cumulative charge of the components is greater than the total charge to patients who are not beneficiaries of a dental benefit plan for the same procedure." o Bundling: "the systematic combining of distinct dental procedures by third-party payers that results in a reduced benefit for the patient/beneficiary." o Upcoding or overcoding: "reporting a more complex and/or higher cost procedure than was actually performed." o Downcoding: "a practice of third-party payers in which the benefit code has been changed to a less complex and/or lower cost procedure than was reported except where delineated in contract agreements." Border molding o Recording of retro mylohyoid area during border molding - muscles in this area are Superior constrictor(directly), medial pterygoid and glossopalatinal(indirectly) o On the buccal of mandibular border molding - u record Buccinator when the pt opens his mouth. When the posterior border(distal of tuberosity) of the max denture is thick, the denture dislodges as the pt opens because the coronoid process hits the denture. If hamular notch hits retromolar pad, what do you need to do before fabricating denture? Intestinal polyps: Gardner's, Peutz Jeghers syndromes.

Cafe au lait spot - Peutz Jeghers syndrome, Albright's syndrome, Basal cell nevus bifid rib syndrome (gorlin - goltz syndrome), Jeff's syndrome (severe form), Neurofibromatosis Peutz Jegher's syndrome inherited intestinal polyps; higher risk for cancer. o S/s: Brownish or bluish-gray pigmented spots, Clubbed fingers or toes, Cramping pain in the belly area, Dark freckles on and around the lips of a newborn, Blood in the stool that can be seen with the naked eye (occasionally), Vomiting o Tx: remove polyps with surgery; Iron supplement o Question on gorlin-goltz and 2 on neurofibromatosis OKC has scalloping of radiolucency around the roots Cemento-osseous dysplasia Black women, middle aged , anterior radioluceny (can be radio opaque); vital; no symptoms A study is designed to determine the relationship bet. emotional stress and ulcers . To do this, the researchers used hospital records of pt's diagnosed with peptic ulcer disease and pt. diagnosed with other disorders over the period of time from july 1988 to july 1998 . The amount of emotional stress each pt. is exposed to was determined from these records. This study is o Cohort, Cross-sectional, Case-study*, Historical Cohort, Clinical Trial Why is the surgical stent required for an immediate denture? to give an idea of the anatomy of the region Which tooth will the matrix band be a problem with when placing a two surface amalgam? o mesial on maxillary first molar b/c of the cusp of carabelli also Mesial Of max premolar> Distal of max molar Asprin stops pain by a. stopping the unpward transduction of pain signal in the spinal cord b.intefere wiht signal intrepretation in the CNS c. stopping the local signal produtction and transduction d.stopping the signal transduction in the cortex Maximum dose of o Prilocaine 400mg METHEMOGLOBINEMIA o Mepivicaine 400mg = 80ml of 0.5% or 40ml of 1% LEAST VASODILATOR EFFECT o lidocaine without epi 300mg (about 9 carpules) o lidocaine with epi 300mg o bupivacaine 90 mg MORE SELECTIVE FOR SENSORY NERVES o *all are amides metabolized in the liver Ethical Principles o Autonomy self-governance duty to respect the patients rights to self-determination and confidentiality o Nonmaleficence do no harm duty to refrain from harming patient o Beneficence do good duty to promote the patients welfare o Justice fairness duty to treat people fairly o Veracity thruthfulness duty to communicate truthfully Which of the following mandibular fracture will heal in 4-6 weeks? Symphysis Question said when does bony healing begin? 1-3 weeks, 4-6 weeks, Mandibular Fractures can always be visualized on pano radiograph; take 2 with another view to confirm o Condylar 29% o Angle 24.5% o Symphysis 22% o Body 16% Pt on kidney dialysis, when can you perform TE? One day after dialysis o Pt with end stage renal disease (ESRD) >>> 1)are on steroid therapy 2)prone to bleeding 3) do NOT use NSAID because they cause nephrotoxicity
nd

Action of Listerine it disrupts adhesion of bacteria to plaque; is a phenolic compound o LISTERINE :Antiseptic mouthrinse is a broad-spectrum antimicrobial, and it kills bacteria associated with plaque and gingivitis by disrupting the bacterial cell wall. Chlorhexidine The mechanism of action is membrane disruption Dental anxiety can be caused by Pts helplessness. What would reduce it? Telling Pt to raise her/his hand when feels pain What does St. John's Wort do? Decrease the body immunity o Note: there is no option anti depressant in choices. o in Pt with HIV it interacts with anti HIV drugs such as Indinavir (increase immunity) and reduces their function so the immunity decreases Primary risk factor for periodontitis? Tobacco o By recent studies, which one has correlation with periodontitis? Diabetes - diabetics are 15 times at risk A kid is on recall appointment and is not cooperative. You should do voice control followed by? Alternating appraisal What is the side effect of pilocarpine (Tx of dry mouth) in toxic dose? Bradicardia and hypotension o Nontoxic side effects: excess sweating and salivation, bronchospasm Which of the following decreases the salivary flow? Scopolamine (like atropine) (also use for nausea and motion sickness) Study group A and B give some agents for plaque control then compare which agent is more effective. Which study is that? o Clinical trial Benzodiazepine works by inhibiting GABA inhibitors Which of the following potentiates anticoagulant activity ? o St. John's Wort o Licorice (Tx of Dyspepsia, indigestion, GERD) and Upper respiratory infections o Saw palmetto Note: never prescribe with coumadine, Increased anticoagulant effect of warfarin has been reported during concurrent ingestion of saw palmetto. Which of the following images best shows the mid-facial fracture? Commuted Tomography (take axial and coronal orientations) o Reverse townes for condyle fracture o Submentovertex for zygomatic fracture o Waters (occipito-mental projection) THE BEST VIEW TO EVALUATE THE FACIAL FRACTURE and maxillary sinus Tx for ClassII furcation involvement (called cul-de-sac)? guided tissue regeneration nd o Note:2 maxillary molar has the worst prognosis in furcation involvement Know about difference between regenerative surgery and flap surgery? o regenerative surgery - for regeneration with bone graft o flap surgery - to get acess for better srp Pt is on rehab of cocaine. what you prescribe for pain? advil Disable Pt comes in and not cooperative, how should you act? Permissiveness (give Pt freedom and treat in the way Pt feel comfortable) EPT not used in primary teeth Ankylosed tooth diagnoses best by Percussion test (metallic sound); See on xray a thick lamina dura and no PDL space

Base metal alloy o Cobalt gives rigidity, strength, hardness o Chromium prevents corrosion o Zinc - increases hardness o Tin helps in porcelain bonding Cheek biting caused by? Decreasing posterior horizontal overlap Also Posterior teeth placed edge to edge. o Tx: reduce facial of mand molar Biting on the corners of the mouth? Reset canine position lip biting may be due to the following: Large anterior horizontal overlap Tongue biting caused by: Having posterior mandibular teeth too lingually (increasing posterior over jet) Two different drugs with same dosages bind to the same receptor and cause same intrinsic affect however they have different affinities for the receptor: In which aspect these 2 drugs are similar? Efficacy, ED50, Potency What reverse alcoholism? Antabuse(disulfiram) it inhibits aldehyde dehydrogenese. o Disulfirum reaction occurs as a result of increased levels of acetaldehyde which gives the hangover feeling. o 510 minutes after alcohol intake, the patient may flushing of the skin, accelerated heart rate, shortness of breath, nausea, vomiting, throbbing headache, visual disturbance, mental confusion, postural fainting, and circulatory collapse. o Metronidazole inhibits this enzyme as well, so dont drink alcohol and take flagyl

After caries removal sound tissue is on cementum. How do you restore? Build up with GI and place composite Periapical lesion caused by all except? Occlusal trauma, Abrasion, Maxillary sinusitis Untreated hyperthyroidism : atropine and excesive amounts of Epi should be avoided Thyrotoxic crisis signs o Early symptoms: restlessness, nausa, abdominal cramp o Later symptoms: high fever, diaphoresis (excess sweating), tachycardia, cardiac decompensation, finally Pt becomes hypotensive. o Note: Betadine Not for use in children under 6 years of age and in patients with a known or suspected iodine hypersensitivity. Regular use is contraindicated in patients and users with thyroid disorders (in particular nodular colloid goitre, endemic goitre and Hashimoto's thyroiditis). Antimicrobial to clean wounds All are the actions of vasoconstrictor in one carpule of local anesthesia: o Increase depth and duration of anesthesia o Reduce systemic absorption of LA What will prohibit mesial drift of tooth toward edentulous area? Proper axio-occlusal contact (opposing and adjacent tooth) Diabetic Pt under NO2 Tx. Precaution? Medication? Change in food? In Neurapraxia which one is affected? Axonal membrane, Perineurium, Nothing is affected o A relatively mild form of nerve injury caused by compression of a nerve. o It involves no structural damage to the nerve axon, although the myelin sheath may be temporarily disrupted. o It is characterized by temporary loss of nerve function, tingling, numbness, and weakness. o It usually heals quickly Least chance of needle injury? cleaning up, recap, setting up Which one is less sedative? promethazine(Phenergan), Benadryl (Diphenhydramine) o Increase in sleep duration chlorpheniramine>promethazine>diphenhydramin. o Phenergan is widely used to treat nausea/vomiting and comes in an injectable, can only be obtained with a prescription. o Benadryl, commonly used for allergies (itchy eyes, runny nose,sneezing) OTC

Radiographic appearance of crater defect? o An osseous crater defect is a concavity in the crest of interdental bone confine within facial and lingual walls Study among smokers and nonsmokers in a period of 6 years (e.g 20002006) to develop disease? Cohort Which race has a higer F in DMFT index? Whites Picture of midline of floor of the mouth,the color is NOT blue o Ranula, dermoid cyst Frey's Syndrome (Auriculotemporal Syndrome) o Neurological disorder that results from injury or surgery near the parotid glands, damaging the facial nerve. o Characterized by flushing or sweating on one side of the face when certain foods are consumed. o Auricutemporal nerve branch of V3, supplyies sensory fibers to preauricular and temporal areas o Carries parasympathetic fibers to the parotid gland. After parotid surgery or trauma the parasympathetic fibers may be severed. o In their attempt to re-establish innervations, these fibers occasionally become misdirected and regenerated along sympathetic nerve pathway. Comparision between 2 proportion, which test we do? o Chi square test measures the association between 2 variables and comparision of groups when the data are expressed as counts or proportions. o T-test camparison of mean value of two groups - difference between 2 means this can be compared between control group vs tx group OR tx A vs Tx B In which syndrome Pt has multiple odontomas? Gardner's syndrome and esophageal stenosis syndrome Which syndrome Pt has calcified falx cerebri, multiple okcs, bifid ribs? - Gorlin Goltz syndrome aka Basal cell bifid rib syndrome. Radiographic projection from the base of the skull: Submentovertex projection o The zygomatic arches stand out like the handles of a jug on this view Which population has the most number of UNRESTORED caries black o Untreated caries blacks o Untreated caries in kids Hispanics o Fillings white A lesion that has re-mineralized will be more difficult to penetrate in the future A class II caries is Apical to contact Caries exists below critical pH of 5.5 What is the bacteria that is not initially involved in caries but plays an important role? LACTOBACILLUS o gram positive facultative anaerobe, the # of this species has been used as a caries test What is Gingival Plaque Index? o Nominal like mild, moderate, severe o Ordinal include numbers: like furcation involvement 1,2,3 o Interval like Celcius degree o Ratio e.g Kelvin degree, or BP measurement(can not be zero), PH, length(can not be negative),weight o Note:gingival index: both ordinal and nominal

Pit and Fissure caries is described as two cones: o Two bases are pointing toward the pulp o Two apexes are pointing toward pulp in smooth surface (proximal caries) o One apex toward the pulp and one base toward DEJ o Both bases facing DEJ How is transillumination helpful when examining a kid? Koplik spots o A prodromic viral enanthem (rash, cluster of spots) of measles manifesting two days before the measles rash itself. o Characterized as clustered, white lesions on the buccal mucosa near each Stenson's duct (opposite the premolars) o Pathognomonic for measles Affected dentin has bacteria present but smaller amounts and can be reversed. Infected dentin has large amounts of bacteria and needs to be removed because non-reversible The dentist accidently did not see Occlusal caries and he placed a sealant over it; Will the caries o Stop o Rapid Progression o Slow Progression What does caries detector stain? Denatured collagen o Research indicates that the dye in the caries detector bonds to the denatured collagen which is present in the outer infected dentin but which is not present in the inner uninfected dentin and normal dentin Recently placed gold inlay; what is the most common reason for pain afterwards? Fracture of the tooth has to be suspected o Galvanic shock Sensitivity - choose this if only question says opposing dissimilar metal For a resin-retained FPD (Maryland Bridge) everything is necessary EXCEPT: o Wrap around proximal o Draw o Occlusal rest o Chamfer o Bevel How is bioavailability measured? o How much drug is absorbed in the circulation o Blood to urine ratio o Note: efficacy of drug: level of binding a drug to its receptor There are two drugs that with the same dosages bind to the same receptor and have same intrinsic affect however different affinities for the receptor: How are these two drugs the same? o ED50 o LD50 o Potency o Efficacy Question asking the definition of antagonist vs. agonist; the answer choices also included partial agonist and partial antagonist o Agonists have intrinsic activity (maximal effect of a drug); efficacy of 1 o Pure antagonists have no intrinsic activity; efficacy of 0 o Partial agonist has an intrinsic value between 0-1 Therapeutic Index = LD50/ED50 or TD/ED and higher therapeutic index is better because wont do as much harm o A drug with high LD50 and low ED50 has high therapeutic index, therefore relatively safe. When you stimulate alpha 1 receptors what happens? Vasoconstriction o Prosazin is a selective 1 blocker used to treat HTN, heart failure, and benign prostate hypertrophy

How will Propranolol (non-selective beta blocker) affect the following? o Ephedrine induced mydriasis (contraction of papillary muscle) ephedrine is Sympathomimetic drug, .with anesthesia causes hypotension. it suppress apetite o Phynylephrine-induced hypertension alpha 1 receptor agonoist Do benzodiazepines have a anxiolytic effect? in moderate doses ANTIANXIOLYTIC and high doses is SEDATIVE Atropine contraindicated for nursing mothers and patients with Glaucoma Zyban (bupropion) is used for smoking cessation. Sodium Thiopental rapid-onset short ultra acting barbiturate(IV) for general anesthesia- for Desensitization What receptors do benzodiazepine act on? GABA Rebound effect when you discontinue med, it brings back the same symptoms that it relieved, stronger than before; Must withdraw gradually to prevent this o Benzos rebound anxiety and insomnia o Eszopiclone and Zolpidem rebound insomnia o Triazolam (short-acting) daytime rebound anxiety, metallic taste, perceptual disturbances o Methylphenidate or Dextroamphetamine Rebound stimulatory effects psychosis, depression and a return of ADHD symptoms o SSRIs rebound depression, anxiety, panic attacks o Clonidine and Guanfacine (alpha-2 adrenergic agents) rebound hypertension What is the most common type of seizures found in children? o Grand mal o Juvenile absence seizures o Simple partial seizure if there was not option of Febrile go with this o Febrile seizure: A convulsion that occurs in association with a rapid increase in body temperature. Common in infants and young children and, fortunately, are usually of no lasting importance. Febrile seizures are the most common type of seizure seen in children. Two to five percent of children have a febrile seizure at some point during their childhood. Tricyclic Antidepressants ( Amitriptyline, Imipramine, Trimipramine) inhibit the reabsorption (reuptake) of serotonin and norepinephrine by brain cells. To a lesser extent, TCAs also inhibit reabsorption of dopamine. Therefore more can reach brain. Lithium- bipolar disorders What do you often need to supplement with diuretics? Potassium Potassium-sparing Diuretics : Spirinolactone, Amiloride, Triamterene Which of the following ACE inhibitor? ends in -PRIL Which of the following drugs causes gingival hyperplasia? Calcium channel blocker : Nephidipine , Cyclosporines Patient comes in and is on Coumadin, what do you do? Do not need to stop medication What drug is used for ANUG? o Tx: topical, then remove pseudomembrane, debride, CHX o Systemic Abts only if there is lymphadenopathy and/or fever Child comes in with an oral infection and is NOT allergic to Pen. What do you prescribe? Penicillin VK What drug has cross allerginicity with Penicillin? Cephalosporin- both have Beta lactamase ring. o If pt has allergic to penicllin then pt has allergy to cephalosporin What is the effect of Tetracycline? BACTERIOSTATIC on protein synthesis 30s What is the effect of Penicillin and Cephalosporin? BACTERIOCIDAL cell wall synthesis Which drug should not be used with someone with Liver damage? Tylenol Effects of histamine and that it is derived from histidine? histamine is bronchospastic and vasodilator o Cimetidine- H2 Blocker (reduce the acid secretion) for GERD (gastro esophageal reflux disease)

Which drug would inactivate the latter? Antacids- Tetracycline o note: Do not take iron supplements, multivitamins, calcium supplements, antacids, or laxatives within 2 hours before or after taking tetracycline. Antacids and milk reduce the absorption of tetracyclines. In a gold MOD onlay, how are the axial pulpal walls? Converging When pouring up a cast which of the following has the least wet ability with a slurry water? o Polyether hydrophilic, no byproduct o Polysulfide byproduct H2o o Condensation Silicone poor wet ability, byproduct ethyl alcohol o Irreversible Hydrocolloid A patient with stable posterior occlusion and only missing maxillary incisors you are making a bridge from #6-#11. What do you need to send to the lab? o Semi adjustable articulator in CR with facebow, lateral records, and incisal guide o Semi adjustable articulator in MIP with facebow, lateral records, and incisal guide o A simple nonadjustable articulator in MIP What is the purpose of a facebow to set the Maxillary arch to the o Terminal hinge axis o Mandibular arch When a person is in physiological rest position freeway space which is 2-3mm Angular chelietis is caused by all of the following except: Increased VDO o Note : Nutritional deficiencies- most common What is the function of Hex on implants? b. anirotational If there is an implant that is 4mm in width at least how many mm does the labiofacial bone need to be? 6mm How does titanium of an implant help in osseointegration? Forms Titanium oxide layer What is the cause of cheek biting in a complete denture? Not enough horizontal overlap A post palatal seal (post dam) is used for Polymerization shrinkage of acrylic Indirect retainer is used to prevent distal extension dislodgement away from the tissue TRUE The flexibility of a clasp is determined by everything except? Depth of undercut What is the strongest type of metal a FPD can be made from? Type 4

"Crown and Bridge" Gold Alloys (Non-ceramic) Type I (soft) - min. 83% Noble Metal Type II (medium) - min. 78% Noble Metal Type III (hard) - min. 78% Noble Metal Type IV (extra hard) - min. 75% Noble Metal Type I - small inlays; very slight stress Type II - inlays, thick 3/4 crowns, complete crowns Type III - thin 3/4 crowns, abutments, pontics, complete crowns, short-span FPD's Type IV - RPD Frameworks, long span FPD's

Kelly (Combination) Syndrome o Destructive changes in hard and soft tissues of patients with complete maxillary denture opposing an unstable bilateral free-end mandibular partial denture o The long-term result is extrusion of the remaining mandibular anterior teeth and the alveolar process surrounding them with loss of posterior mandibular bone. o The plane of occlusion becomes reversed. o Papillary hyperplasia of the hard palate develops. o The premaxilla becomes ATROPHIC as a result of the force exerted on this soft bone during occlusion. o The maxillary tuberosity develops HYPERTROPHY, limited interarch space. o If not corrected, the unstable occlusion can result in progressive posterior mandibular atrophy leading to greenstick fractures. What do a veneer and all ceramic crown have in common? Rounded internal A resin bonded FPD in not seated all the way or is not stable. What could be the reason? microleakage Veneer which is bonded with resin. Patient comes back after a month or so with a dark stain near margin Microleakage When you want to seat a crown, the following contribute to it fitting passively EXCEPT: o Die spacer o Increasing gypsum investment material o Fit checker What is an active screw (post) vs. inactive post? o A smooth-surfaced post although less retentive, transmits the least amount of force to the root structure. Passive, in that they simply lie within the post space after being cemented o Threaded posts actively engage the internal walls of the root canal as they are screwed in The most retentive by far but produces such a force on the brittle root structure that they are contraindicated in most situations Know about metamerism and how it can affect color based on the light source? How do you prevent it? o The quality of some colors that causes them to appear differently under different light sources. o For example, two color samples might appear the same in natural light, but not in artificial light. If a kid comes in and has trauma to the face and the IA is damaged, where did the kid get hit? Angle of mandible o IN KIDS: most mandibular fractures occurred at the condyle (55%), followed by the parasymphysial region (27%), then the body (9%), and angle (8%) o IN ADULTS: Condyle : 29% > Angle 24.5%>Symphysis 22%> Body 16%> Ramus 1.7% > Coronoid 1.3% Know the Maximum dose of lido with epi is 7mg/kg for an adult o Note: text says 5mg/kg for (According to Malamed 4.4mg/kg) and 7mg/kg for articaine Adding o o o o a vasoconstrictor to local anesthesia does all the following EXCEPT: Decreases rate of absorption Increases duration of action Minimizes toxicity and helps homeostasis all of above

Max amount of Nitrous Oxide for a kid o 40 % o 50% o 70% Adult The maximum amount of nitrous on the machine safety hinge is 70% What does band and loop NOT do? Does NOT create a vertical stop Patient with white palate Patient wears denture all night

What is the average pulse rate for a child? 100+ o Newborn to 1yr 100-160 1yr-6yr 60-140 Loss of which tooth important in retaining space Primary second molar

6yr-11yr 70-100

11yr + 60-100

What do you do if mandibular central incisor is erupting lingually? o extract primary teeth and use appliance to move forward o Leave it was an option o note: if primary retain more than 8 yrs of age then extract and place appliance to reposition permanent mandibular anterior Which of the following is most likely to be interpreted as toothache by Pt? o maxillary sinusitis can cause pain or pressure in the maxillary (cheek) area (e.g., toothache, headache) o tmj dysfunction o otitis media

Of the following which is most likely to have reffered pain? o a.acute apical periodontitis o b.irreversible pulpitis o c.phoenix abscess when pulp stone presents on radiograph o a.normal pulp o b.pulp has been injured in the past but has recovered o Some authors believe that pulp calcification is a pathologic process related to various forms of injury, whereas others regard it as a natural phenomenon. lateral periodontal abscess is best differentiated from the acute apical abscess by? pulp testing Radiographically the acute apical abscess o is generally of larger size than other lesions o b.may not be evident o has more diffuse margins than other lesion Epidermology question : o A study is done to determine the affectiveness of a new antihistamine. To do this, 25 allergic pts are assigned to one of the two groups, the new drug (13 pts), placebo (12 pts) . The pts are followed for 6 months . This study is called clinical trail; (assigned or give is the clue ) o Study among smokers and nonsmokers in a persons of 6 years (e.g 2000-2006) to develop disease? cohort Which articular disease most often accompanies Sjgrens syndrome? A. Suppurative arthritis. B. Rheumatoid arthritis. C. Degenerative arthrosis. D. Psoriatic arthritis. E. Lupus arthritis. o May develop years after the onset of an associated rheumatic disorder, such as rheumatoid arthritis, systemic lupus erythematosus, scleroderma, primary biliary cirrhosis etc. (secondary Sjgren's syndrome) Contraindication for max molar with class 2 furcation? hemisection w/ crown o hemisection mand molar Mandibular molars to treat Class II or III furcation invasions o Root amputation max molar What substance has corrosion resistance in pfm metal? Cobalt-chromium and are more rigid in comparison to Ag and Pd o Base metal alloys (non-precious metals) are based on active metallic elements that corrode, but develop corrosion resistance via surface oxidation that produces a thin, tightly adherent film, which inhibits further corrosion. o Base metal alloy advantages are principally found only in their strength and low density.

How does fluoride play role in decreasing caries? makes enamel stronger o The topical effect of constant infusion of a low concentration of fluoride into the oral cavity is thought to increase remineralization of enamel. o Fluoride ion inhibits the enzymatic production of glucosyltransferase inhibits glycolysis o The incorporation of fluoride into the enamel hydroxyapatite crystal producing fluorapatite which is less soluble in catabolic acids produced by oral bacteria. o Prevent caries by directly interfering with the growth and metabolism of organisms such as streptococcus mutans that produce acids responsible for decay. What sleep med do you give to a pt the night before a dental appt? Diazepam? Value is the single most important factor in shade selection. Intensity is included in the term value. o Stains are metallic oxides that fuse to the porcelain during a predetermined firing cycle. o Drastic changes of the hue (color or shade) are often impossible. o Orange stain is the most often used to change the HUE. Staining a porcelain restoration will reduce the VALUE (as will using a complementary color). It is almost impossible to increase the value. o CHROMA can be successfully increased by the use of stains, particularly in the gingival area. Tooth #30 is endo tx with restoration, pt is in pain when he bites, why? cracked tooth Suturing o Resorbable sutures evoke an intense inflammatory reaction. Plain gut or chromic gut are NEVER used for suturing the surface of a skin wound. o When suturing an extraction site in the anticoagulated patient, a non-resorbable suture is recommended. o Suture size is based on strength and diameter. This system uses "0" as the baseline, average size suture. o As suture diameter decreases, "0" are added or numbers followed by a "0" (for example, 000 and 3-0 are the same size). o Because suture material is foreign to the human body, the smallest-diameter suture sufficient to keep the wound closed properly should be used. What meds do you take for asthma? Bronchial relaxation, airway dilation o Beta agonists albuterol, salmeterol metaproterenol What does sodium hypochlorite do? Disinfectant germicidal solvent action o 5.25% solution provides excellent germicidal solvent action What is a sign of ectodermal dysplasia? retained teeth o Hereditary; M>F; from ectoderm; abnormal development of the skin and associated structures (hair, nails, and teeth, and sweat glands o Common clinical findings include hypothrichosis (decrease in hair), anhidrosis (no sweat glands, leading to heat intolerance), anodontia or oligodontia (complete or partial absence of teeth), depressed bridge of nose, lack of salivary glands o child appears much older than what he or she is. All these meds decrease saliva except? Pilocarpine o Pilocarpine (Salogen) is used to stimulate salivary flow in patients suffering from xerostomia due to radiation therapy in the treatment of head and neck cancer. o Pt has round bump on midline of floor of mouth, what is it? ranula o The ranula, a true retention cyst, characteristically occurs in the floor of the mouth and is unilaterally located. Patient is getting front tooth fixed ... whats the purpose of using an incisal guide table? o Acrylic resin reproduces the surface of teeth (lingual concavity/incisal edges) that have a direct influence in guiding the mand. o Through ALL EXCURSIVE movements incisal guidance plays as important a role as the temporo-mandibular articulation in establishing a functional and harmonious occlusion, as much on the anterior teeth as the posterior teeth. Which is least likely to occur with occlusal trauma? gingivitis

On the trauma of occlusion -----gingival reccetion o Radiographic signs of trauma from occlusion: Widening of the periodontal ligament space Sometimes thickening of the lamina dura Angular bone loss and infrabony pocket formation Root resorption Hypercementosis o Trauma from occlusion is reversible, o Periodontal pocket formation is an inflammatory lesions and are not caused by occlusal trauma or bruxism Most common characteristic of cherubism? bilateral jaw swelling Normal occlusal wear shows what? Attrition What substance has corrosion resistance in PFM metal? chromium What sleep med do you give to a pt the night before a dental appt? ambien CR (zolpidem tartrate) 19 yo girl has halatosis, interproximal recession and bleeding ... because? ANUG Best way to suture an incision? interrupted suture Best way to determine outcome of disease? o Med history of the patient (If the lab test was choose may be that) o ESR LAB result Best reason to do an onlay? cuspal coverage What is a sign of ectodermal dysplasia? anodontia, aligodonsia Mechanism of opiods? attach to MU receptors Main advantage of using GI cement? Fluoride Release Why dont you use acidic F with GI? The drug enforcement agency is concerned with what? potential for abuse Where are you most likely to perforate on a maxillary central incisor? Buccal Dentist restored tooth and accidentally left caries ... what happens to caries? caries is arrested Where are the primate spaces? Osteoporosis is associated with which of the following diseases? Hyperparathyroidism Do not give which medication to lactating female? codiene and tetracycline Two things that account for a successful posterior composite restoration? type of resin and type of prep What is the initial step when you have an acute perio abscess? incision and drainage not Abts immediately How do you repair a porcelain veneer with composite? microetch, etch and silane How to do treat porcelain veneer before you cement it?

Which is the best systemic anti-fungal? ketoconazole, and others are amphotericin B and fluconozole b. nystatin c. miconazole d. Clotrimazole conical shaped caries w/ broad base with apex towards pulp is commonly seen in? smooth caries best clinical determinant of root caries a. sensitivity to cold b. sensitivity to sweets c. soft spot on tooth - visual and tactile methods are used for detect caries 40 yo pt w/ all 32 teeth. No cavities. Has stain & catch in pit of molar. what do you do? a. watch and observe b. sealant c. Composite 12 yo kid w/ carious lesion on tooth #9 that shows pulp exposure w open apex... what do you do? a. rct b. apexiflcation What is most numerous at site of chronic inflammation? macrophages Main difference and advantage of using GMT instead of Enamel hatchet? a. bi-angled cutting surface b. angle of the blade c. push/pull action instead of Tooth #30 has huge MOD amalgam and is deep. Hurts pt when he eats french bread. what is the cause? a. root fracture When do you do calcium hydroxide therapy in an avulsed tooth? wait 2 weeks Chronic periodontitis most likely found in?black males What branch off facial nerve gets damaged the most during TMJ surgey? temporal When pt has mouth open, what oral structure would inhibit from capturing buccal flange when taking impression? c. coronoid process Note: Masseter an buccinator are not functioning during opening of mouth. coronoid process has direct involvement in max molar buccal area during mouth opening and can interfere with impression taking. surgical dressing is indicated for what? protect the wound You see 3 mand incisors w/1 wide tooth. x-ray shows tooth has 2 canals. what condition is it? fusion b. Gemination c. Concrescence d. Dilacerations Praising, smiling and congradulating is what type? social reinforcement o Positive reinforcement: Social reinforcement: Praising, smiling, .... No-Social (Token) reinforcement: Toy, Token What do you do when pt on hospital bed has allergic reaction to iv antibiotics? a. take off iv antibiotics b. give epi What are you most likely to see in a kid who has a mandibular canine tipped facially? gingival recession

Why do you do triangular access on incisors (max central inccisor?) a. to help with straight line access

b. help expose pulp horn c. to follow the shape of the crown Metastasis to the oral cavity is most likely to end up where? a.lip b. tongue c. palate d. mucosa e. floor of mouth How do you make a crown narrower? move line angles more facially Not enough room when edentulous pt closes. what do you do? adjust mandibular denture a. tuberosity reduction c. adjust maxillary denture What do x-rays depict when viewing osseous craters ... or something? underestimates bone reduction Unconscious diabetic is treated with what? a. subcutaneous sugar b. give him Orange Juice c. give him insulin injection d. 50 % dextrose solution e. IV sugar solution or IM glucagon pt should wake up within 15mins Most common place of recurrent caries in a class 2 composite? Gingival What type of bond is composite on tooth structure? a. chemical bond b. mechanical bond c. organic coupling d. Adhesion What occlusion class will it be if you have an ANB of -8 degrees? class III Trauma to max incisor, x-ray shows no pulp, asymptomatic and no endo lesion. what do you do? Observe Main advantage of doing direct composite over composite onlay? better marginal adaptation Patient trouble keeping her lower complete denture in because of a low ridge, how to fix this problem? a. ridge augmentation b. place anterior implants c. alveolar bone sectioning Mand complete dentures must be extended how far distally? a. just anterior to retromolar pad b. must sit on the retromolar pad c. must be 5mm beyond the retromolar pad Pt says "your fees seem high" ... how do you respond? "my fees are comparable to geographic area" You are extracting a 2nd mand premolar... what injections do you give? lingual, buccal and IA Most common condition that occurs in the dental office? Syncope What do you do?

Pt has some condition that had blue sclera ... what disease does the pt have? a. osteogenesis imperfect b. marfan's syndrome o A chest that sinks in or sticks out -- funnel chest (pectus excavatum) or pigeon breast (pectus carinatum) Flat feet Highly arched palate and crowded teeth Hypotonia Joints that are too flexible Learning disability Movement of the lens of the eye from its normal position (dislocation) Nearsightedness Small lower jaw (micrognathia) Spine that curves to one side (scoliosis) Thin, narrow face Pt's max denture made her tissue inflamed and weird, you decide to make her a new denture after? a. you reline her old denture a. you place tissue conditioning material in her old denture c. you surgically remove her tissue You inadvertantly extract the wrong tooth ... what do you do? a. replant it asap b. do rct and then replant it c. wait until next appointment to replant d. place implant asap You extract a molar and bone fragments come out with it... what do you do? a. take out bone fragments and make sure its all gone Flap back the tissue and see the entire area Which does not show empathy to the patient? a. open-mindedness b. sharing personal experiences c. reflection and showing understanding Patients with autism will usually show? b. heightened sense of lights and sounds o Children with autism are easily overwhelmed by sensory overload. This can cause stimming (flapping of arms, rocking, screaming, etc). T are hypersensitive to loud noises, sudden movement, and things that are felt. A gingival bevel can be placed with all, except? a. sharp fluted instrument b. enamel hachet c. Gingival margin trimmer d. thin diamiond bur In what situation would a pt need to premedicate? a. mitral valve prolaspe b. prosthetic heart valve c. bicuspid valve disease d. rheumatic heart disease Proximal resistance form of amalgam restoration comes from what? a. convergence of buccal/lingual wall b. retention grooves in axiobuccal/axiolingual walls - for proximal resistance c. Dovetail - provides retention form

Stages of anesthesia o Analgesia /Amnesia o Excitement/Delerium Begins with unconsciousness and ends with loss of eyelid reflex, purposeless movements and hyperreaction, dilated pupils, reflex vomiting, tachycardia and hypertension o Surgical anesthesia working in this stage o Medullary paralysis: Cessation of respiration Denture sore throat : palatoglossal and superior constrictor of pharynx is inflammed (pt. cannot swallow) Port-wine stains associated with sturge-weber Cephalosporins beta-lactam family, anaerobic. A prodrug is a drug made active by metabolism. How much reduction veneer in the middle 1/3 of facial? 0.5mm What does vertical pull headgear with chin cup do? o correct vertical and a wide range of facial myofuntional problems, such as open bite. o Headgear holds the mouth closed continuously, reducing interdental habits and tongue protrusion. o Intrusive force on the molars is increased. o This appliance can quickly change a mouth breathing habit to nose breathing o Know which kind of head gear is for maxillary vs mandible What is an open-ended question? o Allows freedom of response New patient comes into office; 1st visit? o Full exam, probing, med hx, impressions. Why amalgam fails. Bad prep design. What drug causes dilation of eye or midriasis? o Atropine, antimuscarinic How soon do you correct a noticeable crossbite o ASAP

Most difficult part of seting denture teeth? What causes greatest occlusal wear? Porcelain o Amalgam is the strongest to wear What increases in pulp with age? Fibroblasts Width of Keratinized gingiva is : Free gingiva + attached gingiva Bone density changes : Subtraction Radiography Occlusal sealants succeed by change Pt susceptibility Which of material cause less allergy? 1-Lido 2-Procaine 3- epi The most common between five? 1-Papilloma 2-Rhabdomyoma 3-Leiomyoma 4-Lymphangioma 5-Neurofibroma The Most connective tissue tumor : Fibroma (Not true tumor)

TMJ pain are mostly related to: 1- VII

2- V3

3- V2

4-VIII

What is the most common site of new cases of oral cancer? 1-Palate 2-Tongue 3-Floor of mouth 4-lip Major mechanisms for the destruction of osseointegration are o Related to surgical technique o Similar to those of natural teeth o related to implant material o related to nutrition A painless, well-circumscribed radiolucency and radioopacity in the posterior mandible of 11yrs old boy . what is the differential diagnosis Ameloblastic fibro -odontoma (If age above 50 ,its Paget's disease also remember Ameloblastoma occurs in oldies) "Ghost teeth " Regional odontodysplasia (only one side of mouth affected) Where do we find most caries? 1- occlusal2- Proximal Dermoid cyst Occurs in the midline in the floor of the mouth The Stafne defect is a depression of the mandible on the side nearest the tongue. It was previously known as a latent bone cyst and static bone cyst but is now known as a pseudocyst. The depression allows for the presence of a salivary gland Which of the following resembles an epiphysial growth plates? Synchodrosis Most beneficial aspect to brush tongue is reduce? Halotosis What is diff of 330 and 255 bur? 255 is longer The main reason of breaking of RPD clasp? High Mudule of Elastisity Work Hardening HIV with recurrent herpes labialis? Acyclovir

Which medication is controbuting to Insomnia, lack of apetite, and abdominal pain? Dextroamphethamine o Stimulant that promotes NE release in CNS; used for narcolepsy, ADHD, How far the brush and floss goes in sulcus? Brush 1mm , floss 2mm Is Propoxyphene (opiod) contraindicated in pregnant women? In 3rd trimester; WAS WITHDRAWN FROM MARKET What is the best to clean implant? a-water pick b-tooth pick c-floss What is hypertelorism? eyes too far. Seen in Gorlin and Down syndrome What to give to pt with allergy to codeine? Propoxyphene WITHDRAWN FROM MARKET What is 4 -7 years old afraid of ? a- pain b-unknown c-dental chai r d- sepration from parent d-proxibrush

ADHD is most common in? boy Boys have higher rates of ADHD than do girls. Former smoker has less chance of periodontitis compare with current smoker.

Examination reveals a soft, fluctuant, tender swelling in the middle of the hard palate. The teeth test vital. Radiographs reveal a radiolucent area projected between the roots of the maxillary central incisors. The cyst that represents the most likely diagnosis is a o Nasopalatinal cyst o Nasiolabial cyst o Pleomorphic adenoma o globulomaxillary cyst The pulpal floor is perforated during access preparation. The best course of action is to o CONTINUE RCT, REPAIR THE PERFORATION AT A SUBSEQUENT APPT IF ASSOCIATED PATHOSIS DEVELOPS. o REPAIR THE PERFORATION, SCHEDULE PT FOR ANOTHER APPOINTMENT TO FINISH RCT o REPAIR PERFORATION, INITIAL CLEANING/SHAPING SCHEDULE PT FOR FINISHING RCT

Factors that affect the prognosis of perforation repair include o location of perforation o time delay before perforation repair o ability to seal the defect o previous contamination with microorganisms Immediate repair is better than delayed repair, because delay can cause breakdown of the periodontium, resulting in endoperiodontal lesions that are difficult to manage, and elimination of microbial contamination of the defect and proper sealing are critical to success ----------------------------------------------------------------------------------------------------------- Tx of ANUG: ultrasonic debridment, oral rinse chlorhexidine or hydrogen peroxide, o abt tetra and metro only if lymphadenopathy involved o tx of juvenile periodontitis: abt alone or with sc/rp Which of the following drugs best reverses the effects of benzodiazepines? o Naloxone opiod reversal agent o Flumazenil o Midazolam bdz o Aminophylline relaxes respiratory smooth muscle o Physostigmine anticholinergic toxicity antidote; parasympathomimetic inhibits Ach-ase

Following flap surgery, new junctional epithelium can form on either cementum or dentin. Junctional epithelium is reestablished as early as one week. o Both statements are TRUE. o Both statements are FALSE. o The first statement is TRUE, the second is FALSE. o The first statement is FALSE, the second is TRUE Ostectomy is a procedure that involves the use of an autograft. use of an allograft. use of a contiguous graft. removal of tooth-supporting bone. Which of the following drugs is administered orally to treat vaginal candidiasis? Fluconazole (Diflucan) Griseofulvin (Grifulvin) Clotrimazole (Mycelex Troche) Miconazole (Monistat) Nystatin (Mycostatin

Test Cavity o The test cavity method for assessing pulp vitality is very seldom used today. o Used only when all other test methods are deemed impossible or the results of the other tests are inconclusive ex. when the tooth suspected of having pulpal disease has a full coverage crown. o If no sound tooth structure is available to use a bridging technique with the electric pulp tester and cold test results are inconclusive, a small class I cavity preparation is made through the occlusal surface of the crown. o The patient is not anesthetized while this procedure is performed o If the patient feels pain once the bur contacts sound dentin, the procedure is terminated and prep is restored. This signifies that there is some viable nerve tissue remaining, not that the pulp is totally healthy. o If the patient fails to feel any sensation when the bur reaches the dentin, it is a good indication that the pulp is necrotic and root canal therapy is indicated. Treacher Collins syndrome o genetic disorder downward slanting eyes, micrognathia, conductive hearing loss, underdeveloped zygoma, drooping part of the lateral lower eyelids, and malformed or absent ears. Turner tooth----Infection and trauma Hand-Schuller-Christian triad o Diabetes insipidus o Exophthalmos o Bone lesions (Langerhans dis) Oral signs of hand-schuler-christ. = bad breath, sore mouth, loose teeth o lesion are sharply punched out radiolucency and teeth appear as FLOATING IN AIR Amelobelastoma and myxoma---- Hony comb-soap bubble Paget's: Billateral maxilla------Cotton wool Gorlin syndrome or Basal Nevus Cell Syndrome: Bifid rib, OKC, BCC Gardner Syndrome: multiple facial osteoma, Odontomas, hyperdontia, GI polyps w/ potential for colon carcinoma Erythema Multiforme: young men, viral or drug, sudden onset, vermilion, intraoral not on gingiva, (target) bulls eye on hands and feet o Stevens Johnson = severe form of Erythema Multiforme (affects eyes, mouth, and genitalia) PDL widening: Hyperparathyoid, osteosarcoma and scleroderma (Trismus, widened PDL spaces, mask-like face, Excess type I & III collagen) Eagle Syn: Stylohyoid ligament calcification Zoster = shingles = unilateral Multiple neuromas on lips, tongue or palate may indicate that pt has MEN III adrenal pheochromocitoma Tumors of Endocrine glands esp. Medullary carcinoma of thyroid (can cause death) Actinic chelitis---> SCC Oral hairy leukoplakia & Mono, Burkit lymphoma all caused by EBV; nasopharyngeal carcinoma Kaposi sarcoma by herpes 8 Garre's (prolifrative periostitis) and Ewing sarcoma are both onion skin Desquamative gingivitis includes pemphigoid, pemphigus and erosive lichen planus

To change from long scale intensity (low contrast) to short scale intensity (high contrast) but maintain image density, the operator should decrease kVp and increase mAs Auriculotemporal syndrome (Frey syndrome) Often after parotid gland surgery Periapical Cementoosseous Dysplasia OR periapicalcemental dysplasia OR periapical cementoma: o This is a relatively common phenomenon that occurs at the apex of vital teeth. A biopsy is unnecessary because the condition is usually diagnostic by clinical and radiographic features. o Women, especially black women, are affected; appears in middle age (around 40 years) . o The anterior mandible is far more commonly affected than other areas. More often, the apices of two or more teeth are affected. o No Tx Glossodynia: tender or painful tongue Glossopyrosis: burning sensation of tongue Cementoblastoma: o also known as true cementoma, is a rare benign neoplasm of cementoblast origin. o It occurs typically before 25 years of age; no gender predilection. o It is more often seen in the posterior mandible o It is intimately associated with the root of a tooth, and the tooth remains vital. o Cementoblastoma may cause cortical expansion and, occasionally, low-grade intermittent pain. o opaque lesion, usually surrounded by a radiolucent ring representing the periodontal ligament space o TX: cannot be removed without sacrificing the tooth. Bone relief is typically required to remove this well-circumscribed mass. Recurrence is not seen. Warthin's Tumor : Swelling in neck , more diffused Infectious Mononucleosis : Swelling in neck, more rounded and localized Osteosarcoma : Radiographic sunburst appearance Fibrous displysia: Ground Glass Appearance Candidiasis Pt has both burning sensation of tongue and altered taste sensation. Sjogrens syndrome does NOT have burning sensation of tongue but has altered taste sensation. o Pt complains of difficulty wearing the denture and shows cervical caries. o Tx of sicca (dry mouth, eye) in Sjogren syn is pilocarpine Multiple myeloma: Bence jones proteinuria, punched out lucencies 8-bit digital image would have 256 shades of gray Most common salivary gland benign major or minor : Pleomorphic adenoma Most common malignant minor :Adenoid cystic carcinoma Most common malignant major: Mucoepidermoid carcinoma Recurrance: OKC, Ranula

I HAD MANY QUESTIONS ON THIS STUFF: Symptomatic Irreversible Pulpitis o intermittent or spontaneous pain, temperature changes o Advanced thickening of the periodontal ligament o remains untreated the tooth will eventually succumb to necrosis Asymptomatic Irreversible Pulpitis o Left untreated, the tooth may become symptomatic or even necrotic. o Endo treatment should be performed as soon as possible o Internal resorption and pulp polyp are examples. Necrosis o will not respond to electric pulp tests or to cold stimulation. o Pulpal necrosis may be partial or complete and it may not involve all of the canals in a multirooted tooth confusing symptoms, o After the pulp becomes necrotic, bacterial growth can be sustained within the canal. o infection extends into pdl space may become percussion + or exhibit spontaneous pain. o thickening of the periodontal ligament space to the appearance of a periapical radiolucent lesion. o The tooth may become very hypersensitive to heat relieved by cold.. acute periradicular periodontitis o very painful response to biting pressure or percussion; widened PDL chronic periradicular periodontitis o no clinical symptoms; will exhibit a periradicular radiolucency o This tooth is generally not sensitive to biting pressure but can feel different to the patient upon percussion. acute periradicular abscess o very painful to biting pressure, percussion, and palpation. o will not respond to any pulp vitality tests and will exhibit varying degrees of mobility, o widened periodontal ligament space or periradicular radiolucency. o Swelling will be present in the mucobuccal fold and facial tissues adjacent to the tooth. o The patient will frequently be febrile, and the cervical and submandibular lymph nodes will be tender to palpation. chronic periradicular abscess (suppurative periradicular periodontitis) o will not generally present with clinical symptoms. o tooth will not respond to pulp vitality tests and the radiograph or image will exhibit a periradicular radiolucency. o generally not sensitive to biting pressure but can feel different to the patient upon percussion. o will exhibit intermittent drainage through an associated sinus tract. Haloperidol is used for the tx of schizophrenia/tourette syndrome lesion that occurs from tooth flexure? abfraction what liquid is found in glass ionomer? polyacrylic acid infection from mandibular premolars most like to drain into? Sublingual area o mandibular Molars submandibular area The lingual wall of mandibular teeth is most easily perforated when preparing an access opening due to the lingual inclination of these teeth. implant placement is placed where? lmm apical to adjacent cej Where are the primate spaces? max= between lateral & canine & mand= between canine & 1st molar osteoporosis is associated with which of the following diseases? Hyperparathyroidism do not give which medication to lactating female? codiene o This medication may be harmful to an unborn baby, and could cause breathing problems or addiction/withdrawal symptoms in a newborn.

The significant advantage of a nickel-titanium file is its unique ability to negotiate curvatures during continuous rotation without undergoing the permanent deformation or failure that stainless steel files might incur. o A significant risk during NiTi rotary instrumentation is instrument separation. When the instruments are stressed over time, the crystalline structure can change or deform making the files weaker and more prone to reaching their elastic limit. you separate an endo file 3mm from the apex and obturate above it... which case will show the best prognosis? o vital pulp w/ no periapical lesion o vital pulp wI periapical lesion o necrotic pulp wI no periapical lesion o necrotic pulp wI periapical lesion

Least effective sedative? o diphenhydramine (benedryl) sedating-type antihistamines; causes significant xerostomia o nitrous oxide o N2O (weak anesthetic is used agents, such as thiopental, to produce surgical anesthesia) o Diphenhydramine (Benadryl) is representative of the Which is the best systemic anti-fungal? ketoconazole (yes) Conical shaped caries w/ broad base with apex towards pulp is commonly seen in? o root caries (same as smooth caries) o smooth caries (look like v w/ apex towards pulp) o pit/fissure caries (inverted v with apex towards occlusal) Best clinical determinant of root caries o The most commonly used clinical signs to describe root caries utilized visual and tactile (surface texture) o There are no reported clinical symptoms of root caries although pain may be present in advanced lesions. 12 ylo kid w/ carious lesion on tooth #9 that shows pulp exposure wI open apex... what do you do? apexiflcation How enough space provided for eruption of permanent anterior mandible? primate space Dentist charge for crown is $500. Insurance only covers $400. Dentist waves copayment($100) but still tells insurance he charges $500 for crown. what this action called? o a.Down codding o Over billing o c.Price fixing o d.Unbundling How many canal 2nd max primary molar has? 4 o Max first molar 4 canals o Mand first molar 3 canals (25% 4 canals) o Mand second molar 3 canal (25% 4 Canals) Early childhood caries is highest in? b. Hispanic Pt with cafe au lait spots and some problem with iris (lisch nodules)...does not mention about GI polyps o a.neurofibromatosis neurofibromatosis characteristics: lisch nodules: traslucent brown-pigmented spot of iris axillary freckling (crowe's sign), enlargement of fungiform papilla, enlargement of mandibular foramen o b.peutz jegher o c.jaffe syndrome Dentist reviewed of his existing 1000 Pt charts. Noticed among these 1000, last year 200 had periodontitis but in a year after 300 have periodontitis, how much is the incidence? o a.0.3 o b.0.1 o c. 0.2

what kind of study is above? descriptive differential diagnosis of periodontal abscess and periradiculal abscess? o a.percussion o b.vitality test o c.palpation implant success rate after 10 yrs a. 85% is 5 years b.95% c 80%

what percentage of US population have access to fluoridate water? a.45% b.65% (42 states) c.85% You are the 8th dentist, pt did not like none of the previous. Likes you and will bring all his family. Pt suffers from? o paranoid--no trust characterized by paranoia and a pervasive, long-standing suspiciousness and generalized mistrust of others o schizoid personality disorder characterized by a lack of interest in social relationships, a tendency towards a solitary lifestyle, secretiveness, and emotional coldness What is most odontogenic cyst that could end up to ameloblastoma? Dentigerous A primordial cyst is a devleopmental odontogenic cyst. It is found in an area where a tooth should have formed but is missing. Primordial cysts most commonly arise in the area ofmandibular third molars. o Under microscopes, the cyst looks like an odontogenic keratocyst. Kid 5 years with systemic ds came to control plaque- sodium fluoride or chlohexidine Adrenergic receptors o Eye Only alpha1 o Blood vessel Alpha1,2, B2 o Lung Only B2 o Heart B2, B1 Papillion-Lefevre: o Autosomal recessive, impaired T and B lymphocyte o reduced killing of bacterial and fungal infection o advanced periodontitis in primary and permanent dentition related to AA o hemoraghic gingivitis-teeth float in the soft tissue, excessive bone loss, spontaneously tooth exfoliation, palmarplanar keratosis, keratosis of elbow and knee, may be confused with psoriasis o ectopic calcification of falx cerebri. o Tx: rigorous OH, chlorhexidine rinse and periodic antibiotic therapy Mucopolysaccharidosis (autosomal recessive) such as Hurler and Hunter syndrome: o Macroglossia, anterior gingival hyperplasia due to mouth breathing o Thin enamel with pointed cusp in posterior teeth. o Multiple impacted teeth in one large follicle forming a rosette-like pattern radiographically(in Hunter syndrome). o Coronary heart disease (Huler) A practitioner has been using bisect technique for taking xray for his patients. His distance was 8 inches using that system now if he wants to take x-ray utilizing parallel technique form a distance of 16 inches the time of exposure would be: ( double,half.) Same Best caries preventive measure for retained root under Compete Denture cast coping Rectal bleeding, granolumatous gingivitis and recurrent aphtous ulcer. Crohn disease

liner in the casting ring! Its purpose? compensate for casting material expansion Most common cell in necrotic pulp? PMN cells All causes gingival hypertrophy except Diltiazem (phenytoin), dilantin, nifedipine, dixosin, cephalox Medicare is a federal thing that provide health care for elderly . It does not cover dental (both statements are true) Which pulp horn in primary teeth is most susceptible for exposure during operative procedure MB of 2 molar Most common cause of arch space lost in primary dentition decay, ectopic eruption 1 molar Cancer of the other part of the body metastases most commonly to? o Bone:Molar region of mandible o Soft tissue: attached gingiva then tongue o Breast cancer is the most metastatic to the oral cavity appearing in the mandible o then lung cancer is the second appearing in the soft tissue. Picture of a narrow Lucent canal next to upper lateral nutrient cancal? A question pointing to the Ear lobe on the Pano How tooth decay develops fructan has adhesive property then it sticks to the tooth surface After trying the anteriori teeth in the mouth for complete denture patient. In order to determine condylar inclination we make a record of patients anterior guidance. What we have to take i nto consideration in the lab: o Raise the pin on the articulator while setting the condylar inclination A tooth is not responsive to cold, not percussion and pulpation tender necrotic pulp and chronic apical periodontitis A child is allergic to ampicillin, what is the regimen for prophylaxis? ( know the dosage for cephalexin , azithromicin) open tray technique impression for implant to transfer the exact angle of implant to the lab In office bleaching changes the shade through all except surface demineralization After implant placement, an edentulous patient should a. avoid wearing anything for 2 weeks b. immediately have healing abutments placed over the implants ? c. should wear an immediate denture to protect the implant site All implant supported overdenture: Minimum 4 implants in Mand, 6 implants Maxilla o Implant and tissue supported overdenture: For mandibular denture 2 implants in Mand symphysis btw mental foramina, may be joined by a bar o A minimum of 3 months healing time is usually recommended following fixture placement (nonimmediate loading) to allow for osseointegration of the implant fixture what is the most definite way to distinguish ameloblastoma from OKC o a.smear cytology b.reactive light microscopy c.reflective microscopy histologically loss of retepegs is seen in pemphigoid o lichen planus(sawtooth-shaped rete pegs of epithelium)
st nd

Which of the following describes the character of dentinal tubules at the pulpal end when compared to those at the enamel end? a. More per unit surface area and more wider in diameter. b. Less per unit surface area but much wider in diameter. c. More per unit surface area and smaller in diameter d. Less per unit surface area and smaller in diameter For internal bleaching: use sodium perborate & 10% carbamide peroxide****

Have lot of incisal overlap, what do you want to change to maintain balanced occlusion a) condylar inclination b) condylar inclination During the child's first visit, the dentist requested that the parents wait in the reception room. The child cried moderately, but tearfully, throughout the dental examination and prophylaxis. The dentist "gave her permission" to cry while he/she worked and then took no notice of her crying. Her crying diminished in intensity over time and then stopped. With respect ONLY to the crying behavior, the dentist has o used positive reinforcement. o used negative reinforcement. o extinguished the behavior. o ignored the problem.

You have HIV+ pt you can do all of the following except a) treat with metronidozole b) free gingival graft c)prophylaxis to treat candidiasis You give the nitroglycerin to the pt with angina and heart rate goes up what's the reason? o natural reflex to the decrease in blood pressure Which drug will be used to treat an overdose of methotrexate? leucovorine Which fluoride causes the most staining? Stannous New FMX Radio pregnancy or 6 months ago had Taken one

Which kind of caries has been reduced over time? Occlusal, root, proximal Max ridge in denture pt becomes .. over time? Narrow, wide , small , large Diabetic undergoing surgery? o Take Clear fluids with same insulin intake o Take Clear fluids with amount insulin? Pic of dorsum of tongue with 1 side totally red with white nasty patches and painful? Phemigus what sound altered with altered vertical dimension? S sound symptoms if too much codeine? Diabetes leads to Blindness Hep A is transmitted by? Ass to mouth

Stucture most likely damaged with gingival graft? o Greater palatine nerve (I think) o Less palatine nerve Know the level CD4 and neutrophils of HIV patients

If theres space, what is most likely prevent the tooth distal from it to move into space? Occluding to opp tooth? Most associated with developing stage of gingivitis? Macrophages Know that PDL thickens as you get older Mouth breathers leads to? Open bite long face Primary tooth most like to have high chance of pulp exposure? Mand first molar Who has highest autoimmune diseases? Adult female Which diseases are assoc with positive nikolsky sign? Pemphigus If do class 2 on patient with high caries? Resin bond composite If have large caries near pulp, what do you do? Placed GI liner, restore with composite What is function of reciprocal clasp? Retention Oppose force on buccal clasp or stability Signs of hyperocclusion Recession Pain when biting down hard Root resorption Alteration of lamina dura and pdl space Patient with past heart attack, high bp, high chlosterol, asa type? Type 3 Bennett shift, what is the path of movement laterally Distobuccal flange of denture is determined by MASSETER When do the max centals calcify? 2-3 months when does plaque accumulate after eating? 1 hour Know perio diagnosis, what is considered hopeless? Mobility of tooth Our office is fee schedule and pt needs new crown but pt used up all of her allowance (or something like that), what do you do? o Charge same fee Want to compare 2 groups of people, male and female for something, what test do you look at? Chi square test What least likely treatment for class 3 furcation? Extraction What is gives a higher failure rate after perio treatment? Smoking Is tetracycline good for prophylaxis for children? No

Know wrought wire clasp (pg 330 mosby) Know all properties of porcelain Pic of all max anterior lingual totally eroded but incisal edges are fine Pt sucking on lemon Inform consent is for which ethic principle? AUTONOMY

Anesthesia done into the vein results in? o increased BP o increased HR o pain in the periphery

Pt with hemoglobin A1C of 12%,Pt just visited the MD, what kind of TX we can do? Consult with MD prior tx o In most labs, the normal range is 4-5.9 %. o In poorly controlled diabetes, its 8.0% or above o in well controlled patients it's less than 7.0%. Buccal cheek of 60 yrs man, not wipe-able? o leukoplakia( more on floor 50%, tounge25%) o candida o white spongy nevous bilatral- autosomal dominant Temp that necrosis happens during implant placement is 47 for 1-5 min What do u use to drill for implant space o high torque slow speed drill, and saline Green stain in porcelain due to Copper Methotrexate drug interaction beta-lactamase GIC- root caries What do you do when you see a decalcification spot? Fluoride and leave Community water fluoride level 1ppm (1 mg per one liter) Kid is 15 yrs old and living in a community with 0.75ppm of fluoride, what do u do? no supplementation TFO on implant, all of the following except gingivitis and periodontitis Free gingival graft receives its eptithelium from adjacent tissue (blood supply from CT) Where is apical positioned graft contraindicated maxillary palatal region Most common reason for cardiac arrest in children respiratory depression, CHD, atrial septal defects, congenital heart disease

Most common impacted tooth Max canine, max lateral, mand premolar or max premolar(if wisdom was chioce that one) What is the most common tooth to erupt in crossbite? What do u do in a composite restoration. just prep carious pits and fissures Radiograph showing radiolucency in furcation area in a primary second mand molar reason is periodontal, pulp necrosis, trauma If you have to extract 1st, 2nd and 3rd molar, in which order do u extract? 3, 2,1 for better visibility In which direction do you luxate distoangular max third molar distal palatal, distobuccal ,palatal, mesial

What is the diff between LED curing and conventional choices were too long (it is an except question) Rapid Cure - light instantaneously on with no warm up required No heating of substrates - cold curing close up to cure head One peak wavelength of output power (multiple peaks possible as required by process) No fuming of cured material normally caused by excessive radiant power from conventional lamps Safe operation - no UVB, no ozone, no bulb break, low voltage, no mercury Predictable radiant output power with limited drop off with life Variable power - ability to create a cure profile with time e.g. reduced stress curing Even intensity of radiant power over curing area Long life time> 10,000 hrs of LED on time (LEDs only need to be on when curing) Ability to create three dimensional flood areas for curing complex shape What is compomer combined benefits of composites (the comp in their name) and glass ionomers (omer). Why dont you use GI resin cement in cementation of all ceramic restoration - its expansion could cause cracking of porclain Epidermolysis bullosa Rare genetic disorder caused by a mutation in the keratin gene. Characterized by the presence of extremely fragile skin and recurrent blister formation, resulting from minor mechanical friction or trauma. PAN to identify pterygomaxillary fissure Stafne radiographic identification Which cyst does not appear in radiograph Nasolabial Pear shaped Radiolucency on # 9 incisive foramen Which is the best technique to view maxillary sinus options were both MRI and Waters view Best radiograph to view maxillay sinus disease if they are asking about the anatomic form then CT scan is best. If you are intending to find soft tissue pathology then MRI. Best radiograph: CT>waters Best technuque to maxillary sinus: CT>Water> Best for diagnosis for soft tissue: MRI Faint radiolucency with radiopaque border in max sinus Mucocoele U shaped radio-opacity/radioopaque in max sinus zygomatic process Pt has a chronic periapical abscess with sinus tract tx is RCT, what is that tx for sinus tract : antibiotics, curettage, cautery, no tx Which is the best test EPT or thermal cold is best Benign lesion with Cauliflower appearance papilloma Perineural invasion is seen in adenoid cystic carcinoma, Pleomorphic adenoma or low grade muco epidermoid carcinoma This tumor has a marked tendency to invade nerves. Perineural invasion is seen in about 80% of all specimens. Post herpetic neuralgia cause by varicela zoster virus- shigles extreme unilateral pain Why do you give methadone to pt who is trying to stop heroine? to decrease withdrawl symptoms Patient tried using nicotine patches, gum but still couldnt stop what do you do? Buproprion or behavior counseling

When RPD is rocking on the fulcrum line, whats the problem? No option about the base problem with denture base problem with indirect retainer Nitrous oxide safe mechanism level 70%; contraindicated in respiratory infections contraindication: contagious disease, emphysema, bronchitis, autistism, 1st trimester of Pregnancy. Periostat doxycycline 20mg placed in periodontal pocket what does it do? I think it inhibits collagenase What is allograft? same species Couple of questions about bisphosphanate Avoid invasive dental procedures while receiving bisphosphonate treatment. For patients who develop osteonecrosis of the jaw while on bisphosphonate therapy, dental surgery may exacerbate the condition. Clinical judgment by the treating physician should guide the management plan of each patient based on an individual benefit/risk assessment. Dental infections should be managed aggressively and nonsurgically (when possible). Endodontic therapy is preferable to extractions; and, when necessary, coronal amputation with root canal therapy on retained roots to avoid the need for extraction. Dentist tells a patient that mercury is toxic and she should replace her amalgams with composite which principle of ethics dentist does not follow? veracity What do you prescribe for pts with ANUG? Tetracycline in immunocompromised; Debridement and mouthwash Hydrogen peroxide Pt with manic depression (bipolar) disorder not willing to get treated for that is now getting dental treatment from you. What do you see in this patient mood swings One of your patient is having a dental problem and treatment that is not under your capability and you are reffering that patient to a specialist, this type of behaviour comes under which of the following codes of ethic nonmaleficience Which of the following causes Cavernous sinus thrombosis A)Subcutaneous Abscess of upper lip b)Subcutaneous abscess of lower anterior region Infections in upper front teeth are within the area of the face known as the "dangerous triangle". The dangerous triangle is visualized by imagining a triangle with the top point about at the bridge of the nose and the two lower points on either corner of the mouth Pt brushes twice a day n also floss but still he has calculus what do u suggest C) demonstration of brushing First upper molar is supra erupted and the lower opposite molar is missing which of the following u will not do? A) up righting the molar B) rotation of molar C) Intrusion In Implant preparation which of the following can be used A)Hydroxyapatitie Irrigant b) High Speed Hand Piece c) Low torque Drill d)Air Coolant. Cast restoration procedure which impression is least used ? Additional, polyether, Irreversable hydrocolloid (alginate), agar Pt has hepatic dysfunction which pain medication can prescribe? a-Oxycodone Highest cleft palate incidence Asian which of the following dental treatment can be done with pts taking bisphosponate A)Alveoloplasty B) endo C) Extraction D) Scaling b-naproxen c-acetaminophen

Confounding In Epidemiology a confounder is: not part of the real association between exposure and disease o predicts disease unequally distributed between exposure groups o A researcher can only control a study or analysis for confounders that are: known, measurable Example: Grey hair predicts heart disease if it is put into a multiple regression model because it is unequally distributed between people who do have heart disease (the elderly) and those who don't (the young). Grey hair confounds thinking about heart disease because it is not a cause of heart disease. Strategies to reduce confounding are: o randomization (aim is random distribution of confounders between study groups) o restriction (restrict entry to study of individuals with confounding factors - risks bias in itself) o matching (of individuals or groups, aim for equal distribution of confounders) o stratification (confounders are distributed evenly within each stratum) o adjustment (usually distorted by choice of standard) o multivariate analysis (only works if you can identify and measure the confounders) Primary stress bearing area in mandible: a-Retomolar pad b-alveolar ridge c-buccal shelf and in case the residual ridge is in good shape it also contributes to primary support. In which of the following muscles you can place the mandibular denture a-lateral pterygoid b-middle pterygoid c- buccinator (Buccal) d-maseter (Distobuccal) picture of diffused pigmentation in anterior max and mad incisors in attached gingiva? Racial pigmentation 5 yrs with systemic disease what MW used for plaque control a-NaF b- Chlorhexidine which of the following is conditioned stimulus a) dentist b)dental chair which of the following are necessary for a test to be accurate a)specificity b)reliability c)validity Most common mid facial fracture is Zygomatic complex fracture Conventional perioperative management of diabetics Hypoglycemia is much more dangerous in a patient rendered unconscious than hyperglycemia Therefore it is safer to err on the side of hyperglycemia in patients undergoing surgery. Nevertheless ketoacidosis must be avoided. For most well controlled Type II diabetics, hypoglycaemia caused by OHAs and insulin pose a higher risk in the perioperative period than hyperglycaemia. "No glucose no insulin" OHAs should be withheld on the morning of surgery and on the evening before. It is the sulphonyureas that cause hypoglycemia. Take special precaution with chlorpropamide (diabenase) and glibenclamide (daonil) as these have a very long duration of action, and should be stopped a couple of days in advance. NBDE Part 2 Day 2 8 yrs old kid with autism Ectodermal dysplasia Asthma Hx of Heat Attack Sjogerns syndrome Pt with no med Hx but has attrition and lichen planus

Mercury poisoning causes what? Loss of hair Eye sight Weight gain o Will cause peripheral neuropathy (paresthesia, burning or itching), skin discoloration, shedding skin (desquamation) and swelling. o What is a subacute sign of Mercury toxicity? (something like that) Why would you remount outside mouth? o T- muscles wont interfere What causes teeth to be green or orange? o Meds o Intrinsic stuff o Trauma o Decay Combination clasp RPD? o Ibar and cast circumferential o T- wroth wire clasp + cast circumferential o Ibar + T bar o Minor + major connector Wroth wire clasp will reduce torque force (used for perio involved teeth), you use wroth wire for distal extension cases on canine and pre. When person has ketone breath? Hyperglycemic Ortho: difference between skeletal crossbite and non-skeletal crossbite? Thumbsucking causes? o Posterior Crossbite, anterior open bite (most common), constriction of maxilla, class II, crossbite When to stop thumb sucking? Intervention by dentist with appliance at age 5-6 if child has not stopped yet Perio: If little attached gingiva is present, what do you not do? GINGIVECTOMY Full flap to exposed a lot of bone o T- modified widmans EXPOSES TOOTH ROOTS AND ALVEOLAR BONE; no apical repositioning of flap o Mucoperiosteal FULL THICKNESS IgG, what does it effect? o IgG can cross the placenta and give humoral immunity to fetus Rubber band if around central incisors to close diastemas, what condition will allow this? o T- no additional adverse effect Path: Most contagious stage of Syphillis? o Stage I o Secondary stage is most contagious o Stage III Which clinical trial where sample size doesnt make a difference? surveillance If you have pt w/ bad oral hygiene, what type of braces? o No tx o Removable o T- limited fixed o Brackets o When noncompliant patient completes ortho, white kind of retainer? Id assume fixed not removable?

Negative architecture q, when interseptal bone is lower than crestal bone? Lesion in the middle of tongue also pt had it on palate before and pt is healthy? o Karposi o T- candidasissounds right o Syphilis If you want to reverse vasoconstriction to get dilation Proponolol with Epi Vasoconstriction caused by? o alpha agonist, Beta antagonist Nitrous sedation contraindications? Nasocongestion Which is antidepressant? o Something pentol Fluexitine (SSRI), Mazindol, Buspirone, Isocabaxazid If your jaw is broken, allow to heal, how will the growth be? o Bilateral o Symmetrical effective o Condylar side 3cm blue on tongue, what is it? o Hemangioma If you are doing a graft mandibular molar and premolar, what interferes? Which can you anesthesize easier? o mental o Palatal o Long buccal Which is will mostly cause implant to fail? o Oblique ligament o Transeptal ligament Why do porcelain break on FPD Flexure of metal base Major wrong thing with bridge abutment not fit What is pontic like M-D convex If you pack bone, how does allograft heal Bone morphongenic protein Allograft Cadaver bone If hyperocclusion what will it cause occlusal tooth wear fractures of teeth/restorations hypersensitivity of teeth during mastication cervical dentin hypersensitivity tooth hypermobility fremitus abfractions vertical bone loss or localized bone destruction (secondary to periodontal disease) masticatory muscle or TMJ pain. Advantage of inlay over amalgam Esthetics, less tooth reduction

Endo- cant find small orifice what dont you not use Apex locator Apexogenesis, which it doesnt do o Increase blood supply/flow o Normal root lengthening o Close apex o Thicken root Did RCT, pt went home, still sensitive, what is most likely? overinstrumentation Diagnosis of periapical chronic abscess Which cant you use for diagnosis???? o Vitality o Thermal Open apex on primary o Use cold test o If crown on tooth use Thermal test Bring pt for trials modeling If a child is told from parent you will feel no pain from filling, what does this cause o Bad relationship between pt and dentist Unrooly child o

Unruly child is someone who does not respect authority and go abide rules---

Tooth fuse or germination which stage o Abnormalities in tooth size and shape occur as a result of disturbances during the morphodifferentiation stage of tooth development. e.g. fusion, germination, concrescence. AI and OI which stage o Histodifferentiation Abnormalities in tooth structure, namely defects in the enamel or dentin layers, result from disruption during the histodifferentiation, apposition, and mineralization stages of tooth development. What best brushing names Sulcular Aspirin? o Platelet aggregation Aspirin is it extrinsic or intrinsic pathway? Arachidonic is a precursor for Prostaglandin

Implant question on Analog o How implant on cast o Impression taking material Internal component purpose--- external hex give antirotation Max RPD needs to fit how Passively Pt wore denture for 15 years then got irritation o Neuroleema o Neurosarcoma Where does the clasp fit? infrabulge

If you have endo abscess but no sinus tract, can puss drain through pdl o Statement true but reason doesnt confirm What material do you use to capture VDO What is the reason you would do MOD onlay vs Amalgam o Better facial contour o Microleakage Open diastemas 8+9, least successful treatment is o Crown o Veneer o Composite What surface build for oseointigration for implant Surface titanium oxide Mand ant teeth picture with opaque line to the apex Nutrient canal When do you use general anesthesia to work o 2 yr multiple filings o Nervous pt (no) o 4 fillings on all molars o Question said something about a 2 year old needing 12 fillings When do you hit non functional cusps o T- retrusion With age what increases? o Hue o Value o Chroma Microcitic something anemia o T- sickle cell If you have #7 implant what do you need for full success o no contact on lateral movements Which is not a nobel metal o Silver o Cobalt 2 pac smoker no bleeding? Multiple restorations end result o Same occlusal scheme as she had in the start Another vasoconstrictor: know alpha blocker, what causes it, or agonist of alpha and beta Whats used for antidepressant Theopentanol Xray put lateral border orbit, out of focus, or lateral wall of max sinus Know what happens with incorrect vertical or horizontal angulation when taking bitewings on a child Corocoid process /x ray If jaw broken, allow it to heal (without taking care of it), what will be affected: subcondylar fracture, symmetrical non symmetrical

3?s definitive Dx, if shit is growing on side of tongue; incisional biopsy, toulidine blue, cytology, which test he say incisional biopsy Implants- most likely to disturb what? Apical, transeptal, oblique ligaments, or overall musculature Large MOD composite, whats disadvantage? Occlusal wear (T) Space maintainer 4 years lose 1 molars, distal shoe, band and loop, lingual arch, 4 y.o. so you put nothing Know all eruption and calcification sequence when use utility gloves? Cleaning instrument Patient goes to periodontist to get flap surgery, who does re-eval? You, perio, both? (both I believe) Gram negative: broad spectrum antibiotic Physical composition of food, which aspect related is carcinogenic? The amount of times you chew? (a choice, no idea) If you have class V lesion and bottom part on cementum? Occlusal you bevel, part on cementum, 90 degree butt joing When can you have unsupported enamel? Class III, IV, facial of I or II Varicose veins (or large?) On tongue in Elderly? Because of HTN = T ORN blood flow, mandible Put pulp cap Whats over it? GI liner PMMA what is activator/stimulator? Patient comes in with bad perio, why Listerine rinse? because it causes plaque destruction Question about doctor touching PMMA, what kind of reaction may he experience Which electron shell has highest power? (f/doutermost shell) Film Fog caused by Ferrule effect Important for crown lengthening Which material in 100% humidity would distort? Reversible hydrocolloid, addition silicone, etc Cement: When doing composite, what inhibits polymerization? Zinc oxide, Eugenol, Water Highest chance of leakage under rubber dam? Holes too wide, Holes too far apart, Too close Alcoholic INR test Coumadin INR test Tx Alcoholism Arabus? Ketone breath Hyperglycemic Which is worst/most uncomfortable for patient to do: Increase VOD, have CR, slightly off, Decrease VOD (in full dentures) causes angular chelitis Pt has dialysis bring them in day after Definitive Dx: Excisional, Incisional, for a 2x2x3 mm lesion, which one
st

Smoker vs non- smokers observing from 2000-2006 its called a Cohort T test average between clinical trial A & B what to do? Standard deviation, chi test,

Day 2: If ext of max molar what type of tissue on root which is embedded in maxillary sinus o Pseudostratified ciliated columnar epithelium What molar has a c shaped canal (xray shows pa of #31 and the roots where convergent) nd o Mand 2 molar with convergent roots Where is it most important area to have thickness on the metal on FPD on connector area o M-D o F-L On crown where do you place retentive groove o Interproximal o Buccal o lingual Pt has had a cleft palate as a child, it was taken care of and at 30 years wants to get implants. o No! never implants with cleft palateSickle cell o HgS Post bilateral collapse- post mand edentuluous ptwas chewing Picture shows bilateral distal extention and max teeth had been chewing down on the edentulous ridge, man edentulous ridge looked white and beat up, question said Which layer on top later of cells has increased.??? o T- Strt corneum o Strt lucidum T/F metropolol and epi is ok together If pt on non-selected b-blocker can you give epi o Yes o No this is right bc a non-selective b blocker is like proponlol. Max canine and mand canine ectopic from facial, what angle classification o T- I o II o III SNA angle 6 o Class II Liver cirrhosis, need full mouth extraction Periodiagnosis, a lot of chronic periodontitis

Full exthow long do you wait for max denture o 1 day o Same day o 1 week o T- 8 week o 16 week What steps would you use for immediate denture Perio steps: SRP, goss debridement, caries. Pt 14 yo ectopic canine but healthy with asthma o What does she need to bring--- inhaler o There is light sulcular patches on palate and can be scraped, also she used inhaler too much during soccer practice-- T- candida Herpangia Light catch with explorer o PRR All roots furca showing, what would you give him o Fluoride for a week o 1 day o Every 3 months Molar Mesial caries in xray, xray shows caries to pulp and occlusal picture shows super white mesial structure o Early decal o T- Usupported enamel 2 molar stain happened b4 extraction of 1 molar o T- stain happened b4 o After o During What do you not use for ext mand premolar o 150 o 151 o 23 o 286 Pt 60yo impact mand molar, tx plan o Advise pt impacted tooth but dont extract SCC on tongue o What you do---- Incisional 4 teeth what worst prognosis for RCT o Canine o Premolar(decalcified canals) If you upright molar a mesially tilted. Distal and inferior, crown moves distraly root stays at same place Implant, pt diabetic, anti depression, HTN, liver trans 2yo, hep c o T- ASA III o ASA II o ASA IV
nd st

Diabetes can you place implant if HbA1c is 8 o I would say--- refer to physician, and no cant place implants Hawley appliance for skeletal or non-skeletal deformities o Correction of skeletal crossbites Internal derangement cause by tear or stretch of ligaments
o The more common is referred to as an internal derangement with reduction; the disc slides into and out of its normal functional position as the jaw opens or closes, causing the popping sound characteristic of TMD. In cases of internal derangement without reduction, the disc is permanently displaced or dislocated to an incorrect position, and the jaw's range of motion is limited.

Erythromycin decreases the P450 metabolism of other drugs Penetration of xrays is influenced by Kv o Decrease source-film distance, increase density Valium causes orofacial clefting Analgesic compound #3 contains 30mg of codeine Ethyl alcohol is the antidote for methanol because it o Inhibits metabolism o Competes for alcohol dehydrogenase o Prevents formation of formaldehyde o Prevents damage to the optic nerve If patient is taking Cimetidine, they can take Allegra (antihistamine) Constitutional symptom chills Belladonna alkaloids and anticholinergics increase intraocular pressure and are CONTRAINDICATED in patients with GLAUCOMA Increase potency of local anesthetics by lengthening the intermediate chain Chloramphenicol causes aplastic anemia Therapeutic index indicates the safety of a drug The most potent antitubercular drug is ISONIAZID Semilunar flap allows you to see the root with the least soft tissue reflection Rifampin is known to cause oral contraceptive failure Digoxin at toxic levels causes ventricular fibrillation Young patients - vervical stains are prominent If you dont use flux, it causes shrinkage porosity Valproic acid spontaneous gingival hemorrhage upon withdrawal Treat TCA overdose with physostigmine Morphine poisoning pinpoint pupils Phenytoin is used to the treatment of epilepsy and arrhythmias Digitalis is used to treat CHF

In patients on Quinidine for atrial fibrillation, use Digitals to prevent ventricular tachyarrythmias Amyl nitrite o inhalation causes Tachycardia Coronary arterial dilation Peripheral arterial dilation Decreases arterial blood pressure o Prolonged use causes methemoglominemia Cardiac glycosides decrease the concentration of K+ in heart muscle Hydralazine acts directly on arterial smooth muscle Schedule II narcotic and antipsychotic neuroleptic analgesia Benzylpenicillin excreted primarily by renal tubular secretion Irreversible anticholinesterase poisoning die because of respiratory failure via paralysis of diaphragm Chlorpromazine is a blocker orthostatic hypotension Lidocaine is less effective topical anesthesia Hydroxyzine (sedative) dry mouth Tramadol centrally-acting analgesic, binds to Mu, inhibits NE and serotonin reuptake Ketorolac analgesic administration by mouth or IM Horizontally impacted teeth are most difficult mandibular impactions LA + sedation of diabetic patient reduce dose of insulin and dont eat o La but no sedation usual dose of insulin and eat as normal

Você também pode gostar